You are on page 1of 68

COLLEGE OF ACCOUNTANCY AND FINANCE Page 1

ACCO 30013 – Accounting for Special Transactions

INSTRUCTIONAL MATERIALS
FOR
ACCOUNTING FOR SPECIAL
TRANSACTIONS
(ACCO 30013)

COMPILED BY:

REYNAN C. MURTOS
Instructor
COLLEGE OF ACCOUNTANCY AND FINANCE Page 2
ACCO 30013 – Accounting for Special Transactions

TABLE OF CONTENTS

OVERVIEW .......................................................................................................................... 3
COURSE OUTCOMES ......................................................................................................... 4
ACCOUNTING FOR PARTNERSHIP ................................................................................... 5
CORPORATE LIQUIDATION.............................................................................................. 25
ACCOUNTING FOR HOME OFFICE, BRANCH AND AGENCY ......................................... 32
REVENUE RECOGNITION: SALE OF GOODS AND SERVICES ...................................... 39
REVENUE RECOGNITION: CONSIGNMENT SALES ........................................................ 52
REVENUE RECOGNITION: INSTALLMENT SALES .......................................................... 55
REVENUE RECOGNITION: FRANCHISING ...................................................................... 59
REVENUE RECOGNITION: LONG-TERM CONSTRUCTION CONTRACT ....................... 63
REFERENCES ................................................................................................................... 68
COLLEGE OF ACCOUNTANCY AND FINANCE Page 3
ACCO 30013 – Accounting for Special Transactions

OVERVIEW

This course deals with specialized accounting problems likely to be encountered by


accountants. The study of the various topics in this course is based upon fundamental
valuation accounting and accounting theory as applied to special income and expense
recognition method and expanded business operations. This course also includes
specialized problems in partnership accounting; corporate liquidation; accounting for home
office, branch and agency; and revenue from contracts with customers (sale of goods and
services, consignment, installment sales, long-term construction contracts, and franchising).
COLLEGE OF ACCOUNTANCY AND FINANCE Page 4
ACCO 30013 – Accounting for Special Transactions

COURSE OUTCOMES

Upon completion of the course, the students will be able to:


a. Apply the law on partnerships and understand the essential elements and general
aspects in accounting for partnership formation, division of profits and losses, change in
ownership, death and retirement of a partner, and liquidation of partnership.
b. Apply the fundamental concepts in accounting for home office, branch, and agency.
c. Apply the fundamental concepts of revenue recognition in sale of goods and services,
consignment sales, installment sales, long-term construction contracts, and franchising.
COLLEGE OF ACCOUNTANCY AND FINANCE Page 5
ACCO 30013 – Accounting for Special Transactions

ACCOUNTING FOR PARTNERSHIP

Learning Objectives

After discussing the topics, the learner will be able to:


• Define different types of partnership and kinds of partners;
• Understand the initial investment valuation and recognition;
• Identify the different method of recording initial investments;
• Journalize the required entries in the books of the partnership;
• Prepare the statement of financial position immediately after formation;
• Recognize and measure partnership income and expenses;
• Identify the different methods of allocating partnership profit and loss;
• Journalize profit or loss distribution to partners;
• Identify errors and misstatements in the financial statements;
• Define and describe partnership dissolution;
• Identify the different reasons of partnership dissolution;
• Account for the admission of a partner by purchase and by investment;
• Determine the partners’ equity upon withdrawal, retirement, death, or incapacity;
• Record the incorporation of a partnership;
• Define and describe partnership liquidation;
• Identify and explain the causes of partnership liquidation;
• Understand the nature of lump-sum and installment liquidation;
• Understand and apply the concepts of marshalling of assets and right of offset;
• Journalize the realization of non-cash assets, payment of liabilities and final settlement to
partners in lump sum and installment liquidation; and
• Prepare statement of liquidation with supporting schedules of cash distribution (periodic
schedule of payment and cash priority program).

Partnership Formation

Partnership – is a contract whereby two or more persons bind themselves to contribute


money, property or industry into a common fund with the intention of dividing profits among
themselves.

Characteristics of a Partnership
• Co-ownership of contributed capital – Assets contributed to the partnership become
assets of the partnership by virtue of its separate legal personality.
• Income Tax – Partnership except General Professional Partnership (like CPAs, lawyers,
etc.) are subject to income tax rate of 30% based on net income.
• Limited Life – A partnership may be dissolved at any time by action of the partners or by
operation of law.
• Legal Entity – A partnership has a legal personality separate and distinct from that each
of the partners.
• Mutual Agency – Any partner may act as an agent of the partnership in conducting its
affairs.
• Mutual Participation in Profit – A partner has the right to share in partnership profits.
• Unlimited Liability – The personal assets of any partner may be used to satisfy the
creditors’ claims in the partnership if the firm’s assets are not enough to settle the
liabilities to outsiders.

Advantages of a Partnership
• It is easy and inexpensive to organize compared with a corporation.
COLLEGE OF ACCOUNTANCY AND FINANCE Page 6
ACCO 30013 – Accounting for Special Transactions

• The unlimited liability of the partners makes it reliable from the point of view of the
creditors.
• The combined personal credit of the partners offers better opportunity for obtaining
additional capital than a sole proprietorship.
• The participation in the business by more than one person makes possible for a closer
supervision of all its activities.
• The direct gain to the partners is an incentive to give close attention to the business.
• The personal element in the character of the partners is retained.

Disadvantages of a Partnership
• The personal liability of a partner for partnership debts deters many from investing
capital in a partnership.
• A partner may be subject to a personal liability for the wrongful acts or omissions of his
associates.
• It is less stable because it can be easily dissolved.
• There is divided authority among partners.
• There is a constant likelihood of dissension and disagreement when each of the partners
has the same authority in the management of the partnership.
• There is difficulty in disposing of interest since no formal established marketplace exists
for the sale of partnership interest.

Types of Partnership

1. As to Activity
a. Trading Partnership – one whose main activity is the manufacture or the purchase
and sale of goods.
b. Non-Trading Partnership – one organized for the purpose of rendering services.

2. As to Object
a. Universal Partnership
• Universal Partnership of all Present Property – one in which the partners
contribute all the properties which actually belong to each of them at the time of
formation. All assets contributed to the partnership and subsequent acquisitions
become common partnership assets.
• Universal Partnership of all Profit – one which comprises all that the partners
may acquire by their industry or work during the existence of the partnership and
the usufruct of movable property or immovable property which each of the
partners may possess at the time of formation. Partnership assets consist of
assets acquired during the life of the partnership and only the usufruct or use of
the assets contributed at the time of formation. The original movable or
immovable property contributed do not become common partnership assets.
b. Particular Partnership – one which has for its object determinate things, their use or
fruits, or a specific undertaking or the exercise of a profession or vocation.

3. As to Liability of Partners
a. General Partnership – one consisting of general partners who are liable prorata and
sometimes solidarily with their separate property for partnership debts.
b. Limited Partnership – one formed by two or more persons having as members one
or more general partners and one or more limited partners who as such are not
bound by the obligations of the partners.
COLLEGE OF ACCOUNTANCY AND FINANCE Page 7
ACCO 30013 – Accounting for Special Transactions

4. As to Duration
a. Partnership at Will – one for which no time is specified and is not formed for a
particular undertaking or venture and which may be terminated any time by mutual
agreement of the partners or by the will of one alone.
b. Partnership with a Fixed Term – one in which the term or period for which the
partnership is to exist is agreed upon or one formed for a particular undertaking and
upon the expiration of that term or completion of the particular undertaking, the
partnership is dissolved unless continued by the partners.

5. As to Representation to Others
a. Ordinary Partnership – one which actually exists among the partners and also as to
third persons.
b. Partnership by Estoppel – one which in reality is not a partnership but is considered
partnership only in relation to those who by their conduct or omission are precluded
to deny or disprove the partnership’s existence.

6. As to Legality of Existence
a. De Jure Partnership – one which has complied with all the requirements for its
establishment.
b. De Facto Partnership – one which has failed to comply with one or more of the legal
requirements for its establishment.

7. As to Publicity
a. Secret Partnership – one wherein the existence of certain persons as partners is
not made known to the public by any of the partners.
b. Open Partnership – one wherein the existence of certain persons as partners is
made known to the public by the members of the firm.

Kinds of Partners

1. As to Contribution
a. Capitalist Partner – one who contributes capital in the form of money or property.
b. Industrial Partner – one who contributes industry, labor, talent, skills or service.
c. Capitalist Industrial Partner – one who contributes money, property and industry.

2. As to Liability
a. General Partner – one whose liability to third persons extends to his separate
(private) property.
b. Limited Partner – one whose liability to third persons is limited only to the extent of
his capital contribution into the partnership.

3. As to Management
a. Managing Partner – one who manages actively the business of the partnership.
b. Silent Partner – one who does not participate in the management of the partnership
affairs.

4. Other Classifications
a. Liquidating Partner – one who takes charge of the winding up of partnership affairs
upon dissolution.
b. Nominal Partner – one who is not really a partner, not being a party to the
partnership agreement but is made liable as a partner for the protection of innocent
third persons.
c. Ostensible Partner – one who takes active part in the management of the
partnership and is known to the public as a partner in the business.
COLLEGE OF ACCOUNTANCY AND FINANCE Page 8
ACCO 30013 – Accounting for Special Transactions

d. Secret Partner – one who takes active part in the management of the business but
whose connection with the partnership in concealed or unknown to the public.
e. Dormant Partner – one who does not take active part in the management of the
business and is not known to the public as a partner. He is both a silent and a secret
partner.

Articles of Co-Partnership

A partnership is created by an oral or a written agreement. Since partnerships are required


to be registered with the Securities and Exchange Commissions (SEC), it is necessary that
the agreement be in writing. In this case, misunderstanding and disputes among partners
relative to the nature and terms of the contract may be avoided or minimized. The written
agreement between or among partners which governs the formation, operation and
dissolution of the partnership is referred to as the “Articles of Co-Partnership”.

The Articles of Co-Partnership normally includes the following:


• Name of the partnership
• Name and address of the partners
• Rights and responsibilities if each partner
• The purpose for which the partnership is formed
• The initial and additional investments to be made by each of the partner
• The withdrawals that maybe made by partners and the limitations, if any
• The profit and loss ratio
• Procedures on dissolving the partnership

Accounting for Partnership

As compared to other forms of business organization, accounting for partnership differs with
regard to capital accounts. In a partnership, there should be as many capital accounts and
as many drawing accounts as there are partners. Example: In ABC Partnership, the partners
are AAA, BBB and CCC. The capital accounts are: AAA Capital, BBB Capital and CCC
Capital. The drawing accounts are: AAA Drawing, BBB Drawing and CCC Drawing.

The transactions affecting the capital and drawing accounts of each partner are:

CAPITAL
Permanent withdrawal (decrease) of capital Original investment by a partner
Share in the partnership profit from
Share in the partnership loss from operations
operations
Debit balance of drawing account closed to
Additional investment by a partner
capital

DRAWING
Personal withdrawal by a partner in Share in partnership profit from operation
anticipation of profits (temporary withdrawal (this may be credited directly to capital)
of capital)
Share in partnership loss from operation (this
may be debited directly to capital)

Aside from the contributions, the partnership may acquire additional financing from its
present partners. Any loan between a partner and the partnership is always accompanied by
a proper loan documentation such as promissory note. A loan from partner is shown as
“Loan Payable, Advances from Partner, Due to Partner” on the partnership books similar to
any other loan. Unless all partners agree otherwise, the partnership is obligated to pay the
COLLEGE OF ACCOUNTANCY AND FINANCE Page 9
ACCO 30013 – Accounting for Special Transactions

individual partner interest on the loan and such interest is reported in the Income Statement
of the partnership as an “Expense”.

On the other hand, the partnership may also lend money to a partner. In this case, the
partnership records a “Loan Receivable, Advances to Partner, Due from Partner”. Again,
unless otherwise agree by the partners, the loan bears interest and such interest is reported
in the Income Statement of the partnership as “Income”.

Partnership can be formed by (at least):


1. One individual and one individual
2. One individual and one sole proprietorship
3. One sole proprietorship and one sole proprietorship

Partners Contributions

Partners may contribute cash, property or industry to the partnership. Assets contributions
are debited to the appropriate asset accounts and credited to the capital accounts of the
partners. Below are the rules:
• If the asset contributed is cash, it is recorded in the partnership books at face value.
• If the assets contributed are in the form of properties or non-cash assets, such are
recorded at agreed values. In the absence of an agreement, such are recorded at fair
market values.
• If the contribution is service, a memorandum entry is prepared.

In some instances, one or two or all of the partners are former sole proprietors who decide to
unite their assets and liabilities to form a stronger enterprise. In such situation, the new
partnership may open a new set of books or may continue using the books of one of the sole
proprietors. If a new set of books will be used, entries are prepared to record the
contributions. However, if the books of one of the sole proprietors are used, the following
procedures shall be followed:
1. Adjust the books of the sole proprietor which shall be used as partnership books.
2. Record the investment of the other partners.

Capital Credit is Different from Capital Contribution

Prior to recording partners’ initial contributions to the partnership, the individual partners first
agree not only on the valuation of asset contributions but also on their capital credit. The
“capital credit” of each partner is the percentage of equity that each of them will have in the
net assets of the newly formed partnership.

Generally, the capital share of a partner is proportionate to his capital contribution. However,
in recognition of intangible factors such as a partner’s special expertise, established clientele
or necessary business connections, partners may agree to a division of capital that is not
proportionate to their capital contributions. This will give rise to allowing “BONUS” on initial
investments.

Partnership Operation

Accounting for partnership operations is essentially the same as accounting for the
operations of any other forms of business organization. Sale of merchandise on account is
debited to Accounts Receivable and credited to Sales. Collection of accounts is debited to
Cash and credited to Accounts Receivable. The purchase of merchandise on account is
recorded by a debit to Purchases and credit to Accounts Payable. Payment of accounts is
debited to Accounts Payable and credited to Cash. Payment of expenses is debited to
Expenses and credited to Cash.
COLLEGE OF ACCOUNTANCY AND FINANCE Page 10
ACCO 30013 – Accounting for Special Transactions

At the end of the accounting period, adjustments are made for merchandise inventory,
accruals, prepayments, provisions for uncollectible accounts and provision for depreciation.
Profit is determined in the usual manner that is by matching revenues and expenses.

Special Problems in Partnership Accounting


1. Closing Entries
2. Distribution of Profits and Losses
3. Preparation of Worksheet/Working Paper
4. Preparation of Financial Statements

Distribution of Profits and Losses

Since a partnership is composed of two or more persons, there should be a clear indication
on how the partnership profit or loss be equitably divided among the partners. The following
are the factors to be considered in establishing a just and fair profit and loss sharing
agreement:
• Services rendered by the partners to the partnership. Salaries are given to partners
proportionate to time devoted to the organization. Those who devote more time should
have greater salary share. (Salaries).
• Amount of capital contributed by the partners. Interest on capital contribution is
allowed to each partner to give recognition to the differences in capital contributed to the
partnership. Thus, a partner with highest capital contribution gets the highest interest
share. (Interest on Capital – Beginning, Ending or Average).
• Entrepreneurship ability or managerial skills of the partners. Bonus is allowed to
partner when the partnership realizes profits in order to give recognition to managerial
skills. (Bonus)

Rules in Dividing Profits and Losses

The following is the list of rules in the division of profits and losses:
1. As to Capitalist Partners
a. Division of Profits
• In accordance with agreement
• In the absence of an agreement, division of profits is in accordance with capital
contribution
b. Division of Losses
• In accordance with agreement
• If only the division of profits is agreed upon, then the division of losses will follow
the same proportion
• In In the absence of an agreement, division of profits is in accordance with capital
contribution

2. As to Industrial Partners
a. Division of Profits
• In accordance with agreement

b. Division of Losses
• In accordance with agreement
• In the absence of an agreement, the industrial partner shall have no share in his
character as an industrial partner
COLLEGE OF ACCOUNTANCY AND FINANCE Page 11
ACCO 30013 – Accounting for Special Transactions

Correction of Errors

The reported net income (loss) of the partnership maybe incorrect due to the possible errors
and/or omissions in the prior year or in the current year. The effect on income tax paid
should also be considered because the net income distributed to partners is net of applicable
tax. It is understood that the tax implications of the corrections are properly accounted for
especially if the partnership is not a general professional partnership.

Below is the summary of the effect of errors/omission to the net income (loss) of the
partnership:
Corrections to Net Income of current year
No Particulars for the errors made in
Prior Year Current Year
1 Unrecorded prepaid expenses - +
2 Unrecorded accrued expenses + -
3 Unrecorded accrued income - +
4 Unrecorded unearned income + -
5 Overstatement of inventories + -
6 Understatement of inventories - +
7 Overstatement of purchases - +
8 Understatement of purchases + -
9 Overstatement of depreciation none +
10 Understatement of depreciation none -

Partnership Dissolution

Dissolution is defined in Article 1825 of the Civil Code of the Philippines as the change in
the relationship of the partners caused by any partner ceasing to be associated in the
carrying out of the business. It refers to the termination of the life of the existing partnership.
The dissolution of the partnership may be followed by:
1. Formation of a new partnership
2. Liquidation

Reasons of Partnership Dissolution


1. Admission of new partner
2. Retirement or withdrawal of a partner
3. Death, incapacity or bankruptcy of a partner
4. Incorporation of a partnership

Changes in Ownership Structure

Accounting for changes in the ownership structure of a partnership is influenced by the legal
concept of dissolution. When there is a change in the ownership structure, the original
partnership is dissolved and often a new partnership is created. The dissolution and
subsequent creation of a new partnership indicates that a new legal entity has been created
and accounting should measure properly the initial contributions of capital being made to the
new partnership. Changes in the ownership structure of the partnership are presumed to be
arm’s-length transactions which reflect the current value of the partnership. Therefore, such
changes may indicate that:
• The existing assets of the original partnership should be revalued.
• Unrecorded intangible assets exist that are allowing bonus to partners.
COLLEGE OF ACCOUNTANCY AND FINANCE Page 12
ACCO 30013 – Accounting for Special Transactions

Assignment of an Interest to a Third Party

A partnership is not dissolved when a partner assigns his interest in the partnership to a third
party because assignment itself does not change the relations among partners. Such
assignment only entitles the assignee to receive the interest of the assigning partner in
future partnership profits and in partnership assets in the event of liquidation. The assignee
does not become a partner and does not obtain the right to share the management of the
partnership.

Since the assignee does not become a partner, the only change required is the transfer of
the capital interest of the assignor to the assignee. The amount of the capital transfer is
equal to the recorded amount of capital at the time of assignment regardless of the
consideration made.

Example: A, B and C are partners in ABC Partnership with capital of P100,000; P150,000
and P50,000 respectively. Partner A assigns his interest to Partner D for a consideration of
P150,000. What is the entry to record the transfer in the partnership books?

A, Capital 100,000
D, Capital 100,000

Take note that the amount of the capital of the assignor during the time of assignment
should be credited to the assignee. Any excess/deficit is a personal gain or loss of the
assignor. In the example, the excess of P50,000 is a personal gain of Partner A and a
personal loss of Partner D.

Dissolution by Change in Ownership Structure

The dissolution of a partnership doesn’t necessarily mean that the partnership must stop in
doing business. The admission of a new partner and the retirement or death of a partner
require the legal dissolution of the old partnership and the creation of a new partnership.
Accounting for this type of dissolution requires the determination of each partner’s capital in
the business.

Admission of a New Partner

New partners are often a primary source of additional capital or needed business expertise.
A new partner may be admitted in an existing partnership with the consent of all the
partners. Upon admission, the original partnership is automatically dissolved and a new one
is formed although the daily operations of the business generally are not affected. A new
partner may be admitted into a partnership by:
• Purchase – purchase of interest from one or more of the old partners.
• Investment – asset contribution to the partnership.

1. Admission of a New Partner by Purchase. The admission of a new partner by


purchase is the acquisition of capital interest directly from one or more of the present
partners provided that it is with the consent of all the partners. The purchase of interest
from one or more of the partners is a personal transaction between the incoming and the
selling partner and not to the partnership. Thus, any gain or loss is recognized by the
selling partner and not the partnership. The amount of capital transferred will be equal to
the book value of the interest sold regardless of the purchase price or consideration. The
purchase price of the interest sold to a new partner may be:
• Equal to the book value of the interest sold
• Less than the book value of the interest sold
COLLEGE OF ACCOUNTANCY AND FINANCE Page 13
ACCO 30013 – Accounting for Special Transactions

• More than the book value of the interest sold

Proforma Entry:
Capital (Old) xxx
Capital (New) xxx
To record the transfer of capital from old to new partner.

However, revaluation of assets of the old partnership is generally undertaken prior to the
admission of new partner. It may be a positive or negative asset revaluation. The effect
of the asset revaluation is carried to the capital accounts of the old partners. The
adjusted capital of the old partners becomes the basis for the interest transferred to the
new partner.

Proforma Entry:
Other Assets xxx
A, Capital xxx
B, Capital xxx
To record the positive asset revaluation.

A, Capital xxx
B, Capital xxx
Other Assets xxx
To record the negative asset revaluation.

Capital (Old) xxx


Capital (New) xxx
To record the transfer of capital from old to new partner.

Kindly note that in case of asset revaluation, there is no gain or loss to be recognized
since the asset revaluation reflected the adjusted capital of the old partners.

2. Admission of a New Partner by Investment. Admission of a new partner by


investment is a transaction between the original partnership and the new partner. The
investment of the new partner increases the assets of the partnership and its total
capital. The entry to record the admission of the new partner depends upon the capital
interest credited to the partner’s account. In determining the capital credit of the partners,
the following should be fully understood:
a. Agreed Capital (AC) – the amount of new capital set by the partners for the
partnership. It may be equal to, more than or less than the total contributions of the
partners, both old and new. It is also known as new firm capital, total capital or
agreed capitalization. Agreed capital may be indicated in the terms of the admission.
If not given, it may be computed as follows:
• Investment of the new partner divided by the new partner’s interest.
• Investment of the old partners divided by the old partners’ interest.

Example: A and B are partners with capital balances of P100,000 and P200,000
respectively. C is to be admitted by investing P50,000 for a ¼ interest. Agreed capital
may be computed as follows:
• 50,000 / ¼ = P200,000
• 300,000 / ¾ = P400,000

b. Contributed Capital – the investment of all the partners, both old and new. Using
the example above, the total contributed capital is P350,000 (300,000 + 50,000).
COLLEGE OF ACCOUNTANCY AND FINANCE Page 14
ACCO 30013 – Accounting for Special Transactions

c. Asset Revaluation – evidence provided by the amount of an investment which


related to the total value of the business or values of the identifiable assets as
determined on an individual basis by appraisal or other valuation technique. The
amount of the asset adjustment may be determined as the difference between the
agreed capital and the total contributed capital. Generally, asset revaluation upon
partnership formation relate to the partners of the old partnership only.

d. Bonus – the transfer of capital from one partner to another. Bonus to old partners is
capital transfer from new partner to the old partners and vice versa. There is a bonus
when capital credit to individual partners do not equal their capital contributions
although the agreed capital is equal to the total contributed capital.

e. Capital Credit – the interest or equity of a partner in the firm. It is computed by


multiplying the agreed capital by the interest of the partners.

Retirement, Withdrawal or Death of a Partner

The retirement, withdrawal or death of a partner results in the legal dissolution of the
partnership. The dissolution does not require termination of the business. The primary
accounting issue is the proper measurement of the retiring or deceased partner’s capital
account. This requires a determination of the fair value of the partnership at the time the
partner retires or dies including the computation of partnership profit since the end of the last
accounting period. The following is the pro-forma computation of the partner’s capital
balance:

Initial Investment/Beginning Capital


+ Additional Investments
- Withdrawals
+ Share in the partnership profits
- Share in the partnership losses
+ Loans and advances to the partnership
- Loans and advances from the partnership
+ Revaluation of assets (positive asset revaluation)
- Revaluation of assets (negative asset revaluation)
Interest upon Retirement, Withdrawal or Death

The interest of the deceased partner is subsequently settled to his estate. On the other
hand, the interest of the retiring partner may be:
• Sold to a new partner (outsider)
• Sold to existing (continuing) partners
• Sold to partnership

1. Sale of Interest to a New Partner (Outsider). With the consent of all remaining
partners, a retiring partner may sell his interest to an outsider. The sale is recorded in the
same manner as admission of a partner by purchase. The partnership recognizes only
the transfer of capital from the retiring partner to the new partner. No gain or loss is
recorded by the partnership as a result of the sale. Only the reclassification of capital
among the partners is recorded in the partnership books.

2. Sale of Interest to Existing (Continuing) Partners. The interest of the retiring partner
may be acquired by one or all of the remaining partners. The sale is recorded in the
same manner as the sale of interest to a new partner. Therefore, the capital or interest of
the retiring partner is transferred to the capital of the remaining partners. Just like the
sale of interest to new partner, only the reclassification of capital among the partners is
recorded in the partnership books.
COLLEGE OF ACCOUNTANCY AND FINANCE Page 15
ACCO 30013 – Accounting for Special Transactions

3. Sale of Interest to the Partnership. A retiring partner may sell his capital interest to the
continuing partners through the partnership. The partnership records the reduction of
total partnership capital and the corresponding reduction of assets which was paid to the
retiring partner. The settlement to the retiring partner is either by:
• Payment of cash
• Transfer of non-cash assets
• Recognition of a liability

The amount to be paid to the retiring partner may be equal to, less than or more than the
book value of his interest. When the amount paid or the retirement price is not equal to
the interest sold, the difference may be treated as bonus or asset revaluation.

Partnership Liquidation

Dissolution is a legal concept indicating a change in the legal relationship between or


among partners. Termination is the end of the normal business operation of the partnership.
Liquidation is the winding up of business affairs. A partnership is liquidated when its
business operations are completely terminated or ended. The partnership assets are sold,
the creditors are paid and any remaining assets are distributed to the partners as return of
their investment.

Reasons of Partnership Liquidation


• Accomplishment of the purpose for which the partnership was organized
• Termination of the period covered by the partnership contract
• Bankruptcy of the partnership
• Mutual agreement among the partners to close the business
• Court decree

Accounting Problems in Partnership Liquidation


• Determination of the profit or loss from the beginning of the current accounting period to
the date of liquidation and the distribution of such profit or loss
• Correction of accounting errors in prior periods
• Closing of partnership books
• Liquidation of the business

Two Concepts in Liquidation

At the point of liquidation, the assets and liabilities of the partnership are directly intertwined
with those of the individual partners because of the unlimited liability of each partner. The
priorities for creditors’ claim against the available assets to pay the partnership’s liabilities
involve two concepts:
1. Marshalling of Assets – involves the order of creditors’ rights in the partnership assets
and the personal assets of the individual partners. The order in which claims against the
partnership assets will be marshalled as follows:
a. Partnership creditors other than partners
b. Partners’ claim other than capital and profit such as loans payable and any accrued
interest payable
c. Partners’ claim to capital and profits to the extent of credit balances in capital
accounts

2. Right of Offset – involves offsetting a deficit in a partner’s capital against the loan
payable to that partner. The effect is that the loans due to partners (credit balance) are
combined with the respective partners’ capital balances.
COLLEGE OF ACCOUNTANCY AND FINANCE Page 16
ACCO 30013 – Accounting for Special Transactions

Types of Partnership Liquidation

1. Lumpsum Liquidation (Liquidation by Totals) – The distribution of cash to the


partners is done only after all the non-cash assets have been realized, the total amount
of gain or loss on realization is known and all liabilities have been paid.

Procedures:
a. Compute the interest of each partner.
b. Sale of Non-Cash Assets and allocation of gain or loss on realization (Add if gain,
Deduct of loss)
c. Payment of Liquidation Expenses
d. Distribution of cash to creditors and partners

NOTE: Cash distribution to partners is made based on partners’ capital balances. In


case of capital deficiency, it is eliminated by additional investment (if solvent) or charging
to other partners (if insolvent). When personal assets exceed personal liabilities, a
partner is SOLVENT. When personal liabilities exceed personal assets, a partner in
INSOLVENT.

2. Installment Liquidation (Piece-Meal Liquidation) – Assets are realized on a piece-


meal basis over an extended or longer period of time and cash is distributed to partners
periodically as it becomes available.

Two Worst-Case Scenarios:


• All non-cash assets are assumed to be completely worthless. Thus, a hypothetical
loss equal to the carrying values of the non-cash assets is assumed to have
occurred.
• If as a result of the first scenario a partner’s capital account is a debit balance, we
assume that such partner is not able to make contributions to the partnership to
eliminate the hypothetical deficit. This assumption is made regardless of the partner’s
personal financial status.

The liquidation procedures in Installment liquidation is the same as the lumpsum


liquidation except:
• Cash is distributed to partners even before fully realizing non-cash assets and
determining total gain or loss on realization.
• Restricted interest shall consist of:
✓ Remaining unsold assets
✓ Newly discovered unrecorded liabilities
✓ Cash withheld for possible expenses
✓ Debit balances in capital
• A schedule of safe payments or a cash priority program is used as basis to direct the
distribution of any available cash.

Schedule of Safe Payment

It indicates how available cash would be distributed to partners. It is based on the


anticipation of all possible liabilities and expenses including those expected to be incurred in
the process of liquidation. The effect of these items on the partnership capital is allocated
among partners according to their profits and loss agreement. The schedule is also based
on the assumption that all non-cash assets will be worthless. The assumed loss is allocated
among partners according to their profit and loss ratio. The allocation of assumed losses
may result to debit balances in partners’ capital and these balances are treated as
uncollectible. The assumed debit balances are allocated to those partners with credit
COLLEGE OF ACCOUNTANCY AND FINANCE Page 17
ACCO 30013 – Accounting for Special Transactions

balances according to their profit and loss ratio. When the allocation of estimated liabilities,
expenses, liquidation losses and debit balances in capital are completed, assets may now
be distributed to partners in amounts equal to the resulting credit capital balances.

Cash Priority Program

Partners may desire to determine in advance as to whom cash distributions shall be made
as cash may become available. The program is prepared prior to liquidation. The steps in
the preparation of the program are as follow:
a. Determine the total partners’ interest.
b. Divide the total partners’ interest by their profit and loss ratio to get each partner’s loss
absorption capacity.
c. After loss absorption balances are determined, allocations may now be made starting
with partner with the highest absorption balance reduced by the next highest.
d. Once the partners’ absorption balances are equal, cash distributions are made in the
profit and loss ratio.

PROBLEMS

1. AAA, BBB and CCC agreed to form a partnership to be known as ABC Partnership.
What are the entries in the partnership books under different assumptions?
a. Each partner invested cash of P100,000 for an equal interest in the partnership.
b. AAA contributed cash of P150,000 and inventories costing P130,000 and with agreed
values of P150,000. BBB contributed cash of P200,000. CCC contributed equipment
costing P170,000 with accumulated depreciation of P25,000 and agreed value of
P150,000.
c. AAA contributed cash of P100,000; Accounts Receivable of P150,000 with Allowance
for Doubtful Accounts of P50,000. BBB contributed equipment valued at P400,000
while CCC is an industrial partner to contribute his special skills and talents to the
partnership.

2. DDD and FFF, both sole proprietors, agreed to form a partnership. Account balances
and the respective agreed values upon formation are:
DDD FFF
Per Books Agreed Per Books Agreed
Cash 150,000 150,000 140,000 140,000
Accounts Receivable 140,000 140,000 135,000 135,000
Allowance for Bad Debts (50,000) (40,000) (30,000) (40,000)
Inventory 135,000 137,000 128,000 130,000
Equipment 300,000 150,000 200,000 175,000
Accumulated Depreciation (60,000) 0 (20,000) 0
Accounts Payable 100,000 100,000 150,000 150,000

What are the entries in the partnership books under different assumptions?
a. The partners decided to use a new set of books.
b. The partners decided to use the books of DDD.
c. The partners decided to use the books of FFF.

3. GGG and HHH agreed to form a partnership. They initially agreed to divide the initial
partnership capital equally even though GGG contributed P500,000 while HHH
contributed P400,000 cash into the partnership. What are the entries to record the
transactions in the books of the partnership?
COLLEGE OF ACCOUNTANCY AND FINANCE Page 18
ACCO 30013 – Accounting for Special Transactions

4. On January 1, 2021, III and JJJ decided to combine their businesses and form a
partnership. Their balance sheet on this date were:
III JJJ
Cash 150,000 75,000
Accounts Receivable 375,000 250,000
Inventory 500,000 400,000
Furniture and Fixture 600,000 180,000
Office Equipment 230,000 50,000
Prepaid Expenses 12,750 6,000
Total Assets 1,867,750 961,000

Accounts Payable 900,000 360,000


Capital 967,750 601,000
Total Liabilities and Capital 1,867,750 961,000

The parties also agreed to have the following adjustments:


• Provide 5% allowance for doubtful accounts on each Accounts Receivable.
• Inventories should be recognized only at 80% of their book values.
• Furniture and Fixture of III is overvalued by P25,000 while the Office Equipment of
JJJ is overvalued by P11,500.
• Prepaid Expenses of P6,000 for III and P2,000 for JJJ is to be recognized.
• Accrued Expenses of P3,000 for III and P1,000 for JJJ is to be recorded.

Determine the following:


a. Entries to record the formation in the books of the partnership (assuming the partners
decided to use a new set of books)
b. Total Assets after formation
c. Total Liabilities after formation
d. Capital Contribution and Capital Credit of III
e. Capital Contribution and Capital Credit of JJJ

5. Use the same information in Number 4 except the parties agreed that JJJ will make
additional cash investment to give her 50% interest in the firm, after making the
adjustments.

Determine the following:


a. Entries to record the formation in the books of the partnership (assuming the partners
decided to use a new set of books)
b. Total Assets after formation
c. Total Liabilities after formation
d. Capital Contribution and Capital Credit of III
e. Capital Contribution and Capital Credit of JJJ

6. Use the same information in Number 4 except the parties agreed that they will make
settlement among themselves to conform to the 60:40 capital and P&L ratio after making
the adjustments.

Determine the following:


a. Entries to record the formation in the books of the partnership (assuming the partners
decided to use a new set of books)
b. Total Assets after formation
c. Total Liabilities after formation
d. Capital Contribution and Capital Credit of III
e. Capital Contribution and Capital Credit of JJJ
COLLEGE OF ACCOUNTANCY AND FINANCE Page 19
ACCO 30013 – Accounting for Special Transactions

7. KKK, LLL, and MMM formed a partnership on March 1, 2020 with the following
investment: KKK – P500,000; LLL – P600,000 and MMM – P650,000. During the year,
the following transactions took place:
Additional Investment: KKK – P150,000; LLL – P50,000 and MMM – P100,000
Withdrawal: KKK – P0; LLL – P150,000 and MMM – P100,000

During the year, the partnership reported a net income of P150,000. The profit and loss
agreement of the partnership contains the following:
• 10% interest is to be paid on that portion of a partner’s ending capital balance in
excess of P500,000.
• Partner LLL is to receive a bonus equal to 10% of partnership income after interest
and salaries.
• Annual salaries of P24,000 and P36,000 are to be paid to Partners KKK and MMM
respectively.
• Balance to be divided equally.

Determine the following:


a. Share of KKK, LLL and MMM in the net income
b. Capital Balances on December 31, 2020

8. The partnership of NNN, OOO and PPP provides that profits are to be divided as follows:
• Partners are to receive a 10% interest on average capital.
• Each partner is to receive an annual salary of P25,000.
• Partner NNN, the managing partner, is to receive a bonus equivalent to 10% of the
net income after interest and salaries.
• Remaining profit is to be divided equally among partners.

The movement of the capital of the partners is summarized below:


NNN OOO PPP
Investments:
January 1 110,000 180,000 150,000
February 1 20,000
March 1 10,000
May 1 50,000
August 1 20,000
September 1 20,000 50,000
November 1 20,000
December 1 30,000 10,000

Withdrawals:
April 1 10,000
August 1 5,000 5,000

The partnership had a net loss of P250,000 for the year 2020.

Determine the following:


a. Share of NNN, OOO and PPP in the net loss
b. Capital Balances on December 31, 2020

9. QQQ, RRR and SSS are partners sharing profits and losses in 3:3:4 ratio. For the year
2020, the partnership books showed a net income of P150,000. It was disclosed
however, that the following errors were made:
COLLEGE OF ACCOUNTANCY AND FINANCE Page 20
ACCO 30013 – Accounting for Special Transactions

2019 2020
Accrued Expenses not recognized 3,000
Expired Portion of Prepaid Insurance not adjusted 2,500
Inventory overstated 5,000
Unrecorded purchases still in transit FOB SP 10,000
Office supplies used not recognized 4,000

Determine the following:


a. Total effects of the errors in the capital of the firm
b. Corrected Net Income in 2020

10. A, B and C are partners in ABC Partnership with capital of P100,000; P150,000 and
P75,000 respectively. Partner D is to be admitted to the partnership by purchasing ½
interest of Partner A. What is the entry to record the admission under different
assumptions?
a. Partner B declined to accept Partner D into the partnership.
b. Partner D paid P50,000 to partner A.
c. Partner D paid P75,000 to partner A.
d. Partner D paid P30,000 to partner A.

11. A, B and C are partners in ABC Partnership with capital of P100,000; P150,000 and
P75,000 respectively. Partner D is to be admitted to the partnership by purchasing ½
interest of Partner A. The assets should be revalued prior to the admission of Partner D.
What is the entry to record the admission under different assumptions?
a. Partner D paid P50,000 to partner A.
b. Partner D paid P75,000 to partner A.
c. Partner D paid P30,000 to partner A.

12. A and B are partners with capital balances of P200,000 and P100,000 respectively. They
share profits and losses equally. C is to be admitted to the partnership. What are the
entries to record the admission under different assumptions?
a. C invested P100,000 for ¼ interest in the agreed capital of P400,000.
b. C invested P100,000 for 1/5 interest in the agreed capital of P400,000.
c. C invested P60,000 for ¼ interest in the agreed capital of P360,000.
d. C invested P100,000 for 1/5 interest in the agreed capital of P500,000.
e. C invested P60,000 for 1/5 interest in the agreed capital of P300,000.

13. A and B are partners with capital balances of P200,000 and P100,000 respectively. They
share profits and losses equally. C is to be admitted to the partnership by contributing an
Equipment with a cost of P150,000 and accumulated depreciation of P100,000 for a 1/3
interest in the partnership. At the date of admission, the equipment has a fair market
value of P75,000. What are the entries to record the admission?

14. TTT and UUU are partners in a manufacturing business and divide profits 50% to TTT
and 50% to UUU. Their capital balances at December 31, 2020 are as follows: TTT
Capital - P180,000 and UUU Capital - P180,000. The partners agree to admit VVV into
the partnership on January 2, 2020. VVV invested P150,000 cash for a 25% interest in
the partnership capital and profits.

Determine the following:


a. Partners’ Capital after admission (Bonus)
b. Entries to record the admission (Bonus)
c. Partners’ Capital after admission (Asset Revaluation)
d. Entries to record the admission (Asset Revaluation)
COLLEGE OF ACCOUNTANCY AND FINANCE Page 21
ACCO 30013 – Accounting for Special Transactions

15. WWW, XXX and YYY are partners in a manufacturing business with the following capital
balance as of December 31, 2020: WWW – P300,000; XXX – P500,000 and YYY –
P200,000. On January 2, 2021, Partner WWW retired in the partnership and received
P310,000.

Determine the following:


a. Partners’ Capital after retirement (Bonus)
b. Entries to record the retirement (Bonus)
c. Partners’ Capital after retirement (Asset Revaluation)
d. Entries to record the retirement (Asset Revaluation)

16. AAA and BBB are partners are partners in a retail business and divide profits 60% to
AAA and 40% to BBB. Their capital balances at December 31, 2020 are as follows: AAA
Capital - P180,000 and BBB Capital - P180,000. The partners agree to admit CCC into
the partnership on January 2, 2021. CCC purchases a one-third interest in the
partnership capital and profits directly from AAA and BBB for P125,000.

Determine the following:


a. Partners’ Capital after admission (Bonus)
b. Entries to record the admission (Bonus)
c. Partners’ Capital after admission (Asset Revaluation)
d. Entries to record the admission (Asset Revaluation)

17. The capital account balances of the partners in DEF Partnership on July 1, 2020 before
closing of net income are as follows: DDD, Capital (20%) – P600,000; EEE, Capital
(30%) – P1,000,000 and FFF, Capital (50%) – 400,000. The partnership reported a profit
of P3,500,000 during the year. On December 31, 2020, FFF withdrew from the
partnership when he was bought-out by his co-partners for P2,480,000 cash. The net
assets of the partnership as of this date approximated their fair values.

Determine the following:


a. Journal entry in the partnership books to record the withdrawal of FFF
b. Assuming FFF retired on December 31, 2020 and it was agreed that he shall receive
P2,480,000 cash from the partnership in settlement of his interest. What is the journal
entry in the partnership books to record the withdrawal of FFF?
c. Assuming FFF retired on December 31, 2020 and they agreed that he shall receive
cash of P2,000,000 and equipment with carrying value of P400,000 and a fair value
of P1,200,000 in settlement of his interest in the partnership. What is the journal entry
in the partnership books to record the withdrawal of FFF?

18. The statement of financial position of GGG, HHH and III on June 30, 2020 when they
decided to liquidate is given below. The partners share profits and losses in the ratio
2:2:1.
Cash 10,000 Accounts Payable 50,800
Other Assets 140,000 Loans Payable – HHH 2,000
Loans Payable – III 3,200
GGG, Capital (40%) 38,000
HHH, Capital (30%) 24,000
III, Capital (30%) 32,000
Total Assets 150,000 Total Liabilities and Capital 150,000

The other assets were sold for P100,000.


COLLEGE OF ACCOUNTANCY AND FINANCE Page 22
ACCO 30013 – Accounting for Special Transactions

Determine the following:


a. Loss on Realization
b. Prepare a Statement of Liquidation
c. How much GGG, HHH and III will receive on the liquidation?

19. The Statement of Financial Position of the partnership of JJJ, KKK and LLL on
December 31, 2020 when the partners decided to liquidate is as follows:
Cash 150,000 Accounts Payable 200,000
Other Assets 500,000 Loans Payable – JJJ 70,000
JJJ, Capital (40%) 200,000
KKK, Capital (20%) 30,000
LLL, Capital (40%) 150,000
Total Assets 650,000 Total Liabilities and Capital 650,000

Cash is realized for Other Assets as follows and any remaining amounts are distributed
to partners:
Month Book Value Proceeds
January 300,000 260,000
February 200,000 230,000

Determine the following:


a. Gain/Loss on Realization for January and February
b. Restricted Interest for January and February
c. Prepare a Statement of Realization with Schedule of Safe Payment and Cash Priority
Program
d. How much JJJ, KKK and LLL will receive from the liquidation?

20. MMM, NNN and OOO are partners. They divide profits and losses 30%, 40% and 30%
respectively. On June 30, 2020, they decided to liquidate the partnership because of
continuous losses in the operation. The following balances of selected accounts were
taken from the books of the partnership on this date:
Accounts Receivable – NNN 25,000
Loans Payable – OOO 9,500
Loans Payable – MMM 28,800
MMM, Capital 120,000
NNN, Capital 90,000
OOO, Capital 75,000

On this date, the partnership’s assets were P534,400 excluding the receivables from
partners and cash of P28,000. The non-cash assets were sold at 70% of book value.
After the realization, the partnership paid all the outside creditors and liquidation
expenses of P12,000 and distribute the remaining cash to the partners.

Determine the following:


a. Loss on Realization
b. Total Cash Available after realization
c. Cash Available to Partners
d. Amount to be received by each partner

21. Partners PPP, QQQ, RRR and SSS, who share profits 5:3:1:1 respectively decided to
liquidate their partnership. Capital balances at this time were: PPP – P160,00, QQQ -
P140,000, RRR - P130,000 and SSS – P70,000. Additional information:
• Cash on Hand at the beginning is P30,000.
COLLEGE OF ACCOUNTANCY AND FINANCE Page 23
ACCO 30013 – Accounting for Special Transactions

• Partnership furniture with a book value of P25,000 is to be taken over by Partner


PPP at a price of P10,000 in full settlement of a loan payable to him.
• Partnership creditors’ claim of P120,000 and liquidation expenses of P3,000 were
paid off. The remaining non-cash assets were sold at 75% of their carrying values.

Determine the following:


a. Loss on Realization
b. Total Cash Available after realization
c. Cash Available to Partners
d. Amount to be received by each partner

22. The partnership of TTT, UUU and VVV is winding up its affairs and submitted the
following trial balance at September 30, 2020:
Cash 55,000
Other Current Assets 360,000
Non-Current Assets 990,000
Loans Receivable – TTT 120,000
Loans Receivable – VVV 75,000
Accounts Payable 465,000
Loans Payable – UUU 70,000
TTT, Capital (40%) 400,000
UUU, Capital (40%) 400,000
VVV, Capital (20%) 265,000
TOTAL 1,600,000 1,600,000

The partners were able to complete the liquidation in two months only and all cash
available for distribution to partners were distributed at the end of each month. Summary
of transactions for the two months follows:
a. 65% of the current assets were realized in the 1st month at 70% of their carrying
values while the balance was realized at a loss of P20,000 in the 2nd month.
b. The non-current assets were realized as follows:
• 50% was purchased by a competitor at a loss of P95,000 in the 1st month.
• The balance was realized at 80% of its carrying value in the 2nd month.
c. Liquidation Expenses paid:
• 1st month – P15,000
• 2nd month – P12,000
d. Payment of liabilities:
• 1st month – P265,000
• 2nd month – P200,000
e. Cash withheld by the partners was P8,000 in the 1st month.

Determine the following:


a. Loss on Realization
b. Cash Available for Distribution (1st month)
c. Cash Restricted (1st month)
d. Amounts received by each partner (1st month)
e. Cash Available for Distribution (2nd month)
f. Amounts received by each partner (2nd month)

23. WWW, XXX and YYY are partners in a construction business. Because of continuous
losses, they decided to liquidate on July 1, 2020. At this time, the partnership had cash
of P320,000, non-cash assets of P2,100,000 excluding receivable from partners. They
have also substantial labilities. The partners’ capital balances, loan balances and P&L
ratio are as follows:
COLLEGE OF ACCOUNTANCY AND FINANCE Page 24
ACCO 30013 – Accounting for Special Transactions

WWW XXX YYY


Capital balances 350,000 285,000 400,000
Receivables from (to) partners (25,000) 45,000 60,000
P&L ratio 40% 30% 30%

The transactions for the moths of July, August and September are summarized below:
Jul Aug Sep
Proceeds on Sale of Assets:
July (BV sold – 70%) 1,360,000
August (BV sold – 20%) 350,000
September (balance) 180,000
Payment of Liabilities 840,000 300,000 balance paid
Cash withheld for future LE 8,000 10,000 -
Actual Liquidation Expenses paid 10,000 12,000 15,000

Determine the following:


a. Loss on Realization
b. Cash Available for Distribution (July-September)
c. Cash Restricted (July-September)
d. Restricted Interest (July-September)
e. Prepare a Statement of Realization with Schedule of Safe Payment and Cash Priority
Program
f. Amount received by each partner

24. AAA, BBB and CCC are partners. They decided to liquidate on June 1, 2020. On this
date, they have non-cash assets P530,000 and liabilities of P250,000 including loan from
M of P40,000. AAA, BBB and CCC have capital balances of P80,000 (25%); P130,000
(25%) and P90,000 (50%) respectively. The partnership paid P10,000 liquidation
expenses. AAA received P32,000 in the final settlement. All partners are solvent.

Determine the following:


a. Gain or loss on realization
b. Proceeds on the Sale of Non-Cash Assets
c. Amount to be received by BBB and CCC

25. DDD, EEE and FFF are partners who share profits and losses 40:30:30 respectively.
Below are the balances before liquidation:
Cash 72,000
Other Assets 350,000
Accounts Payable 120,000
DDD, Capital 140,000
EEE, Capital 90,000
FFF, Capital 72,000

In the first month of liquidation, 60% of the other assets was sold. Liquidation expenses
of P6,000 were paid, liabilities amounting to P32,400 were paid. The partners distributed
cash available after setting aside P5,000 for future liquidation expenses. Partner DDD
received P37,500 in the first month.

Determine the following:


a. Proceeds on the Sale of Asset and Gain or Loss on Realization
b. Cash Restricted
c. Restricted Interest
d. Amount received by each partner
COLLEGE OF ACCOUNTANCY AND FINANCE Page 25
ACCO 30013 – Accounting for Special Transactions

CORPORATE LIQUIDATION

Learning Objectives

After discussing the topics, the learner will be able to:


• Prepare Statement of affairs with deficiency schedule;
• Journalize transfer of assets from financially distressed company to trustee; and
• Prepare Statement of Realization and Liquidation.

Statement of Affairs

It is in effect a statement of financial position from a “quitting concern” point of view. The
assets are reported at their estimated realizable value instead of book value. They are
reported as pledged with certain creditors or free assets available to general creditors. The
liabilities are reported at their balance sheet amounts listed in terms of their rank as
obligations preferred, secured or unsecured.

Information needed:
1. Balance Sheet
2. Supplementary information such as:
a. Estimates and Appraisals from reliable sources
b. Pledges of Assets
c. Obligations that are expected to emerge in the course of liquidation

Assets in the Statement of Affairs:


1. Assets Pledged with Fully Secured Creditors – Assets that have been pledged but
that are expected to be realized at more than the amount of the claims on which they are
pledged.
2. Assets Pledged with Partially Secured Creditors – Assets that have been pledged but
that are expected to be realized at more than the amount of the claims on which they are
pledged.
3. Free or Unpledged Assets – Assets that have not been pledged and are not related to
individual liability terms.

Assets Side Columns:


1. Book Value of Assets
2. Name of Assets
3. Appraised Value of Assets
4. Estimated amount available for unsecured creditors
5. Estimated gain/loss on realization

Liabilities and Equity in the Statement of Affairs:


1. Preferred Creditors – Claims that must be provided in full before anything may be paid
to remaining unsecured claims (by law).
2. Fully Secured Creditors – Claims that have been pledged certain assets that are
expected to realize as much or more than the amount of the claims.
3. Partially Secured Creditors – Claims that have been pledged certain assets that are
expected to realize less than the amount of the claims.
4. Unsecured Creditors – Claims that carries no legal priority and on which there is no
assets pledged.
5. Contingent Liabilities – Any contingent liabilities which are expected to develop into
actual liabilities.
6. Capital – Balances summarizing the interests of owners of the business.
COLLEGE OF ACCOUNTANCY AND FINANCE Page 26
ACCO 30013 – Accounting for Special Transactions

Liabilities and Equity Columns:


1. Book Value of Liabilities and Capital
2. Name of Liabilities and Capital
3. Amount of Unsecured Liability

Procedures in preparing Statement of Affairs:


1. Section Headings should be first be set up.
2. Each liability should be considered and reported in the appropriate liability section.
3. After all liabilities have been considered together with assets pledged on such claims, all
remaining assets represent unpledged items and may be listed as such.
4. Asset and liability data are summarized and the statement is completed.

Deficiency Statement

This statement is prepared to accompany the Statement of Affairs. It summarizes the


sources of deficiency such as:
• Losses on Realization
• Additional liabilities and liquidation expenses
• Losses to be borne by owners

Statement of Realization and Liquidation

It is a summary of the course of operations of a business under the administration of a


trustee and involving the realization of assets and liquidation of obligation.

ASSETS
Assets to be Realized: Assets Realized:
(BV of Non-Cash Assets) (Net proceeds of assets sold)
Assets Acquired: Assets not Realized:
(Additions to assets during liquidation period) (BV of unsold non-cash assets)

LIABILITIES
Liabilities Liquidated: Liabilities to be Liquidated:
(Paid liabilities) (BV of liabilities)
Liabilities not Liquidated: Liabilities Assumed:
(Unpaid liabilities) (Additional obligations incurred during
liquidation period)

REVENUES AND EXPENSES


Supplementary Charges: Supplementary Credits:
(Purchases and expenses) (Sales and other income)

If total debits are greater than total credits, there is loss on realization; however, if total
credits are greater than total debits, there is a gain on realization.

Trusteeship

An independent body (trustee or receiver) is appointed by court to take over the assets of
the insolvent corporation to protect the equities and rights of all parties concerned.

Proforma Entries:

Debtor’s Book:
Trustee’s Account xxx
COLLEGE OF ACCOUNTANCY AND FINANCE Page 27
ACCO 30013 – Accounting for Special Transactions

Allowance for Doubtful Accounts xxx


Accumulated Depreciation xxx
Current Assets xxx
Non-Current Assets xxx
To record the transfer of assets.

Trustee’s Book:
Current Assets xxx
Non-Current Assets xxx
Allowance for Doubtful Accounts xxx
Accumulated Depreciation xxx
Debtor’s Account xxx
To record the receipt of assets.

Note: Only assets are transferred to the books of the trustee. The original liabilities and the
stockholder’s equity accounts are left on the company records.

Proforma Entries (Trustee Books)


Transactions Entry
Accounts Receivable xxx
a Sales on account
Sales xxx
Purchases xxx
b Purchases on account
Accounts Payable xxx
Cash xxx
c Collection of old accounts receivables
Accounts Receivable xxx
Allowance for Bad Debts xxx
Write off of uncollectible old accounts
d Bad Debts Expense xxx
receivable
Accounts Receivable – old xxx
Cash xxx
e Collection of old notes receivable with interest Notes Receivable – old xxx
Interest Revenue xxx
Cash xxx
f Collection of new notes receivable with interest Notes Receivable xxx
Interest Revenue xxx
Interest Expense xxx
Payment of old accounts and notes payable Corporate Liability – AP (old) xxx
g
with interest Corporate Liability – NP (old) xxx
Cash xxx
Interest Expense xxx
Payment of new accounts and notes payable Accounts Payable xxx
h
with interest Notes Payable xxx
Cash xxx
Cash xxx
i Sale of assets with gain Assets xxx
Gain on Sale – Corp. xxx
Cash xxx
j Sale of assets with loss Loss on Sale – Corp. xxx
Assets xxx
Expenses xxx
k Payment of operating expenses
Cash xxx
l Record year end adjusting entries (the usual manner)
m Close nominal accounts to Income Summary (the usual manner)
Income Summary xxx
Debtor’s Account xxx
Close the balance of Income Summary to
n
Retained Earnings
Debtor’s Account xxx
Income Summary xxx
o Close corporation’s liabilities paid accounts Debtor’s Account xxx
COLLEGE OF ACCOUNTANCY AND FINANCE Page 28
ACCO 30013 – Accounting for Special Transactions

Corporate Liability – AP (old) xxx


Corporate Liability – NP (old) xxx

If the trustee is able to restore the financial solvency, the control of the assets is returned to
the former owners, HOWEVER, if solvency cannot be restored, then the corporation will be
liquidated.

PROBLEMS

1. AAA Company is being liquidated. The following information is related to the liquidation:
• Bonds payable amounting to P73,600 is secured by merchandise inventory with a
book value of P123,000 and net realizable value of 2/3 of the recorded amount.
• Of the 195,600 accounts payable, P55,000 is secured by equipment with carrying
value of P76,800 which is 70% realizable.
• Building with carrying value of P129,000 has a net realizable value of P99,000.
• Other recorded liabilities are: accrued interest on bonds – P3,100; salaries payable –
P17,400; taxes payable - P11,600 and liquidation expenses – P8,500.
• Cash available prior to liquidation amounts to P12,000.
• Total assets of the company prior to liquidation amounts to P480,000. Except for
prepaid expenses of P7,600 and goodwill of P22,000 which has no value, the
remaining assets have a net realizable value equivalent to 80% of the recorded
amount.
• Total liabilities of the company prior to liquidation amounts to P380,000.

Determine the following:


a. How are the liabilities classified?
b. Gain/Loss on Realization
c. Amount Available to Unsecured Creditors
d. Percentage of Recovery
e. Amount payable to Partially Secured Creditor

2. The listings of assets and liabilities of BBB Company on June 30, 2020 along with
estimated realizable values are as follows:
BV NRV
Cash 250,000 240,000
Accounts Receivable 630,000 450,000
Inventory 600,000 500,000
Equipment 450,000 150,000
Land and Building 750,000 420,000
Other Assets 30,000 0
Accounts Payable 1,200,000
Notes Payable 300,000
Wages Payable 72,000
Taxes Payable 228,000
Mortgage Payable 615,000
Share Capital 900,000
Retained Earnings (Deficit) (615,000)

Additional Information:
• Accounts Receivable are pledged as security for the notes payable.
• The mortgage payable is secured by the Land and Building.
• Liquidating Expenses are expected to be P35,000.
• Unrecorded Liabilities amounting to P20,000 is to be recognized.
COLLEGE OF ACCOUNTANCY AND FINANCE Page 29
ACCO 30013 – Accounting for Special Transactions

Determine the following:


a. Gain/Loss on Realization
b. Fully Secured Liabilities
c. Partially Secured Liabilities
d. Unsecured Liabilities
e. Amount Available to Unsecured Creditors
f. Percentage of Recovery
g. Amount payable to Partially Secured Creditor
h. Statement of Deficiency

3. The Balance Sheet of CCC Company at June 30, 2020 is as follows:


Cash 350,000
Accounts Receivable 700,000
Inventory 500,000
Prepaid Rent 50,000
Land and Building 2,200,000
Machinery 600,000
Patent 450,000
Total 4,850,000

Accounts Payable 1,150,000


Wages Payable 430,000
Real Estate Tax Payable 100,000
Notes Payable 550,000
Mortgage Payable 1,650,000
Share Capital 2,000,000
Retained Earnings (Deficit) (1,030,000)
Total 4,850,000

Additional Information:
• The company estimated that P630,000 is the maximum amount collectible for the
accounts receivable.
• Except for 20% of the inventory that are damaged and worth only P20,000, the cost
of the other items is expected to be recovered in full.
• The land and building have a net realizable value of P1,700,000 and are subject to
the mortgage payable.
• The appraised value of the machinery is P200,000.

Determine the following:


a. Gain/Loss on realization
b. Amount available to Unsecured Creditors
c. Estimated settlement per peso of unsecured liabilities
d. Amount recoverable by each class of creditors

4. DDD Company provides the following balance sheet as of December 31, 2020:
Current Assets (NRV of P2,500,000) 3,000,000
PPE (NRV of P4,500,000) 7,500,000
Other Assets (NRV P200,000) 650,000
Total 11,150,000

Accounts Payable (secured by Inventories with NRV of P1,600,000) 4,500,000


Loans Payable (secured by PPE with NRV of P2,800,000) 5,000,000
Ordinary Shares 2,500,000
COLLEGE OF ACCOUNTANCY AND FINANCE Page 30
ACCO 30013 – Accounting for Special Transactions

Retained Earnings (Deficit) (850,000)


Total 11,150,000

Determine the following:


a. Amount available to Unsecured Creditors
b. Estimated Deficiency
c. Amount expected to be received by Accounts Payable Creditors

5. The creditors of EEE Company agreed to the following concession in recognition of it’s
deteriorating financial condition:
• ABC Corporation, one of EEE’s suppliers, agreed to accept merchandise at its
normal selling price of P1,350,000 in full satisfaction of P1,458,000 overdue accounts
receivable from EEE. The cost of the merchandise to EEE was P1,100,000. ABC’s
accounts receivable from BSA included a P135,000 allowance for doubtful accounts.
• Philippine Prudential Bank, agreed to accept 2,000 shares of EEE’s P450 par
ordinary shares with a current market price of P800 per share in full satisfaction of
P2,025,000 note and P180,000 accrued interest due from EEE. PPB has provided a
P450,000 allowance for this note.

Determine the total gain or loss resulting from the concession.

6. FFF Company is insolvent and its statement of affairs show:


Estimated gain on realization of assets 2,000,000
Estimated loss on realization of assets 2,560,000
Additional assets 1,200,000
Additional liabilities 960,000
Share Capital 12,000,000
Retained Earnings (Deficit) (11,200,000)

Determine the pro-rata payment to stockholders.

7. Below is the summary accounts appearing in the Statement of Realization and


Liquidation of GGG Company:
Assets to be realized 5,000,000 Liabilities not liquidated 1,900,000
Assets not realized 2,700,000 Liabilities assumed 1,200,000
Assets realized 3,500,000 Liabilities liquidated 2,500,000
Assets acquired 1,800,000 Supplementary charges 850,000
Liabilities to be liquidated 4,250,000 Supplementary credits 600,000

Determine the Gain/Loss on Realization.

8. HHH Company was unable to pay its current obligations as they become due. SM
Company was appointed as trustee on January 2, 2020. Creditors and stockholders
agree that an attempt should be made to rehabilitate the business assets, pay off the
creditors and distribute remaining funds to the stockholders. The trustee is authorized to
take over all the assets of HHH. A statement of financial position was given to the trustee
and the following business transactions were selected.

March 31 – Sales on account, P1,050,000


April 8 – Collections of notes receivable, P75,000
May 15 – Payment of accounts payable old – P150,000

Determine the following:


a. What is the entry for the March 31 transaction in the books of HHH and SM?
COLLEGE OF ACCOUNTANCY AND FINANCE Page 31
ACCO 30013 – Accounting for Special Transactions

b. What is the entry for the April 8 transaction in the books of HHH and SM?
c. What is the entry for the May 15 transaction in the books of HHH and SM?

9. The following data were taken from the statement of realization and liquidation of III
Company for the quarter ended September 30, 2020:
Assets to be realized 750,000 Liabilities not liquidated 1,825,000
Assets not realized ? Liabilities assumed 750,000
Assets realized 875,000 Liabilities liquidated 1,770,000
Assets acquired 880,500 Supplementary charges 620,500
Liabilities to be liquidated 1,500,000 Supplementary credits 845,000

The beginning balance of Share Capital and Retained Earnings are P510,000 and
P148,000 respectively. The net income for the period is P224,500.

Determine the following:


a. Assets not realized
b. Beginning balance of cash
c. Ending balance of cash

10. A statement of realization and liquidation has been prepared for JJJ Company. The
totals are given below:
Assets to be realized 65,000 Liabilities not liquidated 28,500
Assets not realized 80,000 Liabilities assumed 32,000
Assets realized 30,000 Liabilities liquidated 58,000
Assets acquired 48,000 Supplementary charges 120,000
Liabilities to be liquidated 95,000 Supplementary credits 110,000

Retained Earnings increased to P25,000. The beginning balance of Share Capital and
Retained Earnings are P120,000 and (P35,000) respectively.

Determine the following:


a. Beginning balance of cash
b. Ending balance of cash
COLLEGE OF ACCOUNTANCY AND FINANCE Page 32
ACCO 30013 – Accounting for Special Transactions

ACCOUNTING FOR HOME OFFICE, BRANCH AND AGENCY

Learning Objectives

After discussing the topics, the learner will be able to:


• Differentiate the operations of home office, branch, and agency;
• Journalize the transactions of an agency, home office and branch;
• Journalize inter-branch transactions of merchandise transfer (at cost and above cost)
and inter-branch transfer of fixed assets.
• Prepare a reconciliation statement of home office and branch accounts;
• Determine net income of an agency, home office, branch and the consolidated net
income;
• Prepare working paper for the combined financial statements; and
• Prepare consolidated financial statements.

Agency

Agencies are established to display merchandise and to take customers’ order but they don’t
stock merchandise to fill their orders or to pass on customers’ credit. Agencies are given
only samples of merchandise to display. Agencies are not required to have a complete
accounting system. They keep only records of cash receipts and disbursements.

Proforma Entries:
No Transactions Home Office Books
Working Fund – Agency xxx
1 Establishment of Working Fund
Cash xxx
Samples – Agency xxx
2 Shipment of Samples
Shipment to Agency xxx
Accounts Receivable xxx
3 Receipts of orders from customers
Sales – Agency xxx
Cost of Sales xxx
4 Cost of sales associates with agency sales
Shipment to Agency xxx
Expenses - Agency xxx
5 Replenishment of Working Fund
Cash xxx
Expenses – Agency xxx
6 Adjustment of Samples
Samples – Agency xxx
Payment of agency expenses by home Expenses – Agency xxx
7
office Cash xxx
Sales – Agency xxx
Cost of Sales – Agency xxx
8 Closing of Revenues and Expense
Expenses – Agency xxx
Agency Income xxx
Closing of Agency Income to Retained Agency Income xxx
9
Earnings Retained Earnings xxx

Branch

Branches stock merchandise coming from the home office or acquired from other suppliers,
make sale to customers, pass on customer’s credit, collect receivables, incur expenses and
perform other functions normally associated with the operations of a separate entity.
Branches have their own books of accounts and prepare financial statements and submit
them to home office for consolidation.

Proforma Entries:
No Transactions Home Office Branch
1 Cash sent by Home Office Branch xxx Cash xxx
COLLEGE OF ACCOUNTANCY AND FINANCE Page 33
ACCO 30013 – Accounting for Special Transactions

to Branch Cash xxx Home Office xxx


Merchandise sent by Branch xxx Shipment from HO xxx
2
Home Office to Branch Shipment to Branch xxx Home Office xxx
Merchandise returned by Shipment to Branch xxx Home Office xxx
3
Branch to Home Office Branch xxx Shipment from HO xxx
Cash remitted by Branch Cash xxx Home Office xxx
4
to Home Office Branch xxx Cash xxx
Fixed assets purchased by
Home Office for the Fixed Assets xxx
5 Memorandum Entry
Branch (records are kept Cash xxx
by Home Office)
Fixed assets purchased by
Home Office for the Branch xxx Fixed Assets xxx
6
Branch (records are kept Cash xxx Home Office xxx
by Branch)
Adjusting entry for the Branch xxx Depreciation Expense xxx
7
depreciation in No. 5 Accum. Depreciation xxx Home Office xxx
Adjusting entry for the Depreciation Expense xxx
8 No Entry
depreciation in No. 6 Accum. Depreciation xxx
Fixed assets purchased by
Fixed Assets xxx Home Office xxx
9 Branch (records are kept
Branch xxx Cash xxx
by Home Office)
Accounts payable of
Branch xxx Accounts Payable xxx
10 Branch paid by Home
Cash xxx Home Office xxx
Office
Accounts receivable of
Branch xxx Cash xxx
11 Home Office collected by
Accounts Receivable xxx Home Office xxx
Branch
Branch expenses paid by Branch xxx Expenses xxx
12
the Home Office Cash xxx Home Office xxx
Branch expenses charged Branch xxx Expenses xxx
13
by Home Office Expenses xxx Home Office xxx

Closing Entries

At the end of the period, the nominal accounts are closed in the books of both companies.

Proforma Entries:
No Transactions Home Office Branch
Inventory, end xxx Inventory, end xxx
Sales xxx Sales xxx
To close the balance of Shipment to Branch xxx Inventory, beg xxx
1 revenue and expenses Inventory, beg xxx Shipment from HO xxx
accounts Purchases xxx Purchases xxx
Expenses xxx Expenses xxx
Income Summary xxx Income Summary xxx
To close the branch net (already recorded as an Income Summary xxx
2
profit to the HO account adjusting entry) Home Office xxx
Income Tax Expense xxx
3 To record income taxes No Entry
Income Tax Payable xxx
To close the branch
Branch xxx
4 income to Income No Entry
Income Summary xxx
Summary
To close the combined net
Income Summary xxx
5 income to Retained No Entry
Retained Earnings xxx
Earnings
COLLEGE OF ACCOUNTANCY AND FINANCE Page 34
ACCO 30013 – Accounting for Special Transactions

Reconciliation of Home Office and Branch Account

The home office and branch accounts are reciprocal accounts, meaning they should be
equal at all times. However, reciprocity will not exist if errors or omissions have been made
in recording inter-branch transactions, or if transactions have been recorded in the other
books but not in the other.

The reconciliation of the home office and branch accounts is similar to the approach used in
bank reconciliation. The most common reconciling items are:
• Shipment by home office in transit
• Merchandised returned by the branch not yet received by the home office
• Collections by the home office for the account of the branch
• Collections by the branch for the account of the home office
• Expenses allocation of the home office to the branch
• Payment of branch account by the home office
• Payment of home office account by the branch
• Remittance by the branch still in transit

Consolidation of Financial Statements

The financial statements of the branch are submitted to the home office for consolidation
because the home office and the branch are considered as a single entity.

Consolidation Procedures:
1. Combine like items of assets, liabilities, equity, revenues and expenses of the home
office and branch.
2. Eliminate inter-company transactions.
• Eliminate home office and branch account.
Home Office xxx
Branch xxx

• Eliminate merchandise shipment accounts.


Shipment to Branch xxx
Shipment from HO xxx

• Eliminate allowance for overvaluation of inventory.


Allowance for overvaluation xxx
Inventory xxx

• Eliminate intercompany sales and purchases.


Sales xxx
Cost of Sales xxx

• Eliminate intercompany payables and receivables.


Accounts Payable xxx
Accounts Receivable xxx

Shipment to Branch at Above Cost

Proforma Entries:
Home Office Branch
Branch xxx
Shipment from HO xxx
Shipment to Branch xxx
Home Office xxx
Allowance for Overvaluation xxx
COLLEGE OF ACCOUNTANCY AND FINANCE Page 35
ACCO 30013 – Accounting for Special Transactions

The billed price is either:


• Based on sales (gross profit rate)
• Based on cost (mark-up rate)

Inter-Branch Transfer of Merchandise with Excess Freight

There are instances when home office finds it necessary to authorize the transfer of
merchandise from one branch to another branch. Since branches receiving shipments are
properly charged with freight and freight is properly accounted as inventoriable cost, any
excess freight should be absorbed by the home office and treated as an operating
expense. On the other hand, any reduction in freight is treated as savings/income by the
home office.

PROBLEMS

1. AAA Company put up an agency in Sta. Mesa and had the following transactions with
Sta. Mesa agency for the month of January:
• AAA transferred merchandise to Sta. Mesa agency to be used as samples - P13,000
and P10,000 as working fund.
• Receipts of sales order from the agency – P120,000.
• Collection of agency accounts by the home office – P91,000.
• AAA paid P10,500 for agency expenses.
• Replenishment of the agency working fund upon receipt of expense vouchers –
P6,850.
• Cost of goods sold identified with the agency sales – P93,000.
• Appraised value of the samples at the end of the month is estimated to be P8,000.

Determine the following:


a. Net Income (Loss) of the agency
b. Entries to record the transactions in the books of AAA and Sta. Mesa Agency

2. On March 15, 2020, BBB Company established an agency in Quezon City, sending its
merchandise samples costing P15,750 and working fund of P9,000 to be maintained on
the imprest basis. During the month, the agency transmitted to the home office sales
orders which were billed at P64,380 of which P20,400 was collected. A home office
disbursement chargeable to agency is the acquisition of furniture and fixtures amounting
to P25,000 to be depreciated at 24% per annum. The agency paid expenses of P3,815
and received replenishment thereof from the home office. On March 31, 2020, the
agency samples were valued at P10,075. It was estimated that the gross profit on goods
shipped to bill agency sales orders average 25% of cost.

Determine the following:


a. Net Income (Loss) of the agency
b. Entries to record the transactions in the books of BBB and Quezon City Agency

3. Home office and branch accounts of CCC Company show activities for the month of
February:
HOME OFFICE
Cash remitted 42,000 Beginning balance 15,000
Merchandise returned to HO 3,000 Shipment at cost 32,000
Fixed assets charged to HO 5,000 Expense allocated from HO 14,500
HO note collected with int. 2,100
COLLEGE OF ACCOUNTANCY AND FINANCE Page 36
ACCO 30013 – Accounting for Special Transactions

BRANCH
Beginning balance 15,000 Branch remittance 36,000
Shipment at cost 37,000 Fixed asset purchased 5,000
Expense allocated to branch 15,400
Note collected by branch 2,000

Except for a branch error in recording expense allocations and home office error in not
recording interest, all differences in the accounts are due to timing differences in
recording reciprocal information.

Determine the following:


a. Unadjusted balance of Home Office and Branch accounts
b. Adjusted balance of the Home Office and Branch accounts
c. Adjusting entries on both books

4. The unadjusted trial balance of the Home Office and Branch as of December 31, 2020 is
as follows:
Accounts Home Office Branch
Cash 116,000 31,250
Accounts Receivable 104,250 63,000
Inventory 200,600 0
Branch 196,500 0
Furniture and Fixture 100,000 40,000
Accumulated Depreciation 22,000 0
Accounts Payable 112,750 51,250
Home Office 0 196,500
Share Capital 250,000 0
Retained Earnings 302,100 0
Sales 525,000 330,00
Shipment to Branch 251,000 0
Purchases 612,500 112,500
Shipment from HO 0 251,000
Operating Expenses 133,000 80,000
Total 1,462,850 1,462,850 577,750 577,750

Adjustments:
• Depreciation Expense: Home Office – P5,900; Branch – P3,250
• Inventory, end: Home Office – P242,500; Branch – P117,500

Determine the following:


a. Adjusting Entries on both books
b. Closing Entries on both books
c. Elimination Entries
d. Net Income of Home Office
e. Net Income of Branch
f. Consolidated Net Income

5. DDD Company operates a branch in Cebu City. Operating data for the home office and
branch:
Home Office Branch
Sales 300,000 78,500
Purchases from outsiders 210,000 20,000
Shipment to Branch/from HO 30,000 40,000
COLLEGE OF ACCOUNTANCY AND FINANCE Page 37
ACCO 30013 – Accounting for Special Transactions

Expenses 60,000 12,500


Inventory, 01/01/2020
From outsiders 80,000 7,500
From home office 0 24,500
Inventory, 12/31/2020
From outsiders 55,000 5,500
From home office 0 26,000

Determine the following:


a. Inventory on January 1 and December 31 at cost
b. Cost of Sales of the Branch at cost
c. True income of the Branch
d. Consolidated net income

6. The income statement submitted by Branch to the Home Office for the month of January
is as follows:
Sales 600,000
Less: Cost of Sales
Inventory, Jan 1 80,000
Shipment from HO 350,000
Purchases 30,000
Inventory, Jan 31 (100,000) 360,000
Gross Profit 240,000
Less: Operating Expenses 180,000
Net Income (Loss) 60,000

The inventory of the branch consisted of:


January 1 January 31
From home office 70,000 84,000
From outsiders 10,000 16,000

After effecting necessary adjustments, the home office ascertained that the net income
of the branch should be P156,000.

Determine the following:


a. Percentage of billing by home office
b. Allowance for overvaluation of inventory

7. EEE Company has a branch in Davao City. Davao branch receives all its merchandise
from EEE at 25% above cost and sells them at 40% mark-up on cost. EEE also sells
merchandise to outsiders at 40% above cost. Below are the excerpts from the trial
balance of EEE and Davao branch:
Home Office Branch
Sales 5,880,000 3,360,000
Purchases 6,250,000
Inventory, beginning 350,000 200,000
Shipment to Branch 2,000,000
Shipment from HO 2,375,000
Allowance for overvaluation 540,000
Operating Expenses 1,200,000 325,000

Determine the following:


a. Inventory of branch on December 31
COLLEGE OF ACCOUNTANCY AND FINANCE Page 38
ACCO 30013 – Accounting for Special Transactions

b. Consolidated inventory
c. True income of the branch
d. Consolidated Net Income
e. Allowance for Overvaluation

8. FFF Company just opened a branch in Baguio City this February of the current year.
Summary of transactions for the 1st month of operations follows:
• Baguio Branch received P15,750 cash from home office to start its operations.
• Home office shipped merchandise to the branch costing P100,000 at 25% mark-up
on cost, of which 20% have not yet received by the branch.
• Branch purchases from other suppliers amounted to P89,250.
• Branch sales for the month amounted to P245,000.
• Home office debit memo for P3,500 representing the branch’s share on advertising
expenses was recorded only by the branch on the 3rd day of March.
• Branch operating expenses of P61,250 were paid by the home office.
• Branch remitted P29,750 to the home office but the home office recorded it only in
March.
• The inventory on hand of the branch is: from outsider – P21,000 and from HO –
P37,500.

Determine the following:


a. Unadjusted and adjusted balance of Home Office account
b. Unadjusted and adjusted balance of Branch account
c. Net Income reported by the Branch
d. True Income of the Branch

9. The Manila branch of GGG Company is billed for merchandise at 20% gross profit rate.
The branch sells them at 25% above cost. On March 17, the entire branch’s
merchandise was destroyed by fire. The branch records that were recovered show:
Inventory, beginning (at billed price) 165,000
Shipment, Jan 1 – March 17 (at billed price) 100,000
Purchases from outsiders – all were sold at 20% mark-up on cost 7,500
Sales 169,000
Sales returns 3,750

Determine the following:


a. Branch beginning inventory at cost
b. Cost of merchandise destroyed by fire

10. The Home Office shipped merchandise to Baguio Branch costing P10,000 and paid a
freight of P650. Baguio Branch was subsequently instructed to transfer merchandise to
Laoag Branch wherein Baguio Branch paid P200 freight. If the shipment was directly
shipped from Home Office to Laoag Branch, the freight cost would have amounted to
P700.

Determine the following:


a. Entries to record the transactions
b. Entries to record the transactions assuming the freight from Home Office to Laoag is
P1,000.
COLLEGE OF ACCOUNTANCY AND FINANCE Page 39
ACCO 30013 – Accounting for Special Transactions

REVENUE RECOGNITION: SALE OF GOODS AND SERVICES

Learning Objectives

After discussing the topics, the learner will be able to:


• Apply PFRS 15 in accounting revenue from contract with customer;
• Know the criteria in identifying a contract;
• Be able to identify performance obligations in a contract;
• Differentiate performance obligations satisfied over time from performance obligations
satisfied point in time;
• Be able to determine the transaction price and allocate to the performance obligations;
• Be able to account for variable considerations; and
• Be able to account for contract modifications.

PFRS 15: Revenue from Contracts with Customers

Effectivity - for annual periods beginning on or after January 1, 2018, with early adoption
permitted.

It supersedes:
• PAS 18 Revenue;
• PAS 11 Construction contracts;
• IFRIC 13 Customer Loyalty Programmes;
• IFRIC 15 Agreements for the Construction of Real Estate;
• IFRIC 18 Transfers of Assets from Customers; and
• SIC-31 Revenue – Barter Transactions Involving Advertising Services.

Objective: to establish the principles that an entity shall apply to report useful information to
users of financial statements about the nature, amount, timing and uncertainty of revenue
and cash flows arising from a contract with a customer.

Scope

The scope of PFRS 15 applies to all contracts, provided the counterparty to the contract is a
customer, subject to these specific exceptions:
1. Lease contracts (PAS 17 Leases)
2. Insurance contracts (PFRS 4 Insurance Contracts)
3. Financial instruments and other contractual rights or obligations within the scope of
PFRS 9 Financial Instruments (PAS 39 Financial Instruments: Recognition and
Measurement), PFRS 10 Consolidated Financial Statements, PFRS 11 Joint
Arrangements, PAS 27 Separate Financial Statements and PAS 28 Investments in
Associates and Joint Ventures
4. Non-monetary exchanges between entities in the same line of business to facilitate
sales to customers or potential customers. For example, PFRS 15 does not apply to a
contract between two oil companies that agree to an exchange of oil to fulfil demand
from their customers in different specified locations on a timely basis.

Customer - a party that has contracted with an entity to obtain goods or services that are an
output of the entity’s ordinary activities in exchange for consideration
COLLEGE OF ACCOUNTANCY AND FINANCE Page 40
ACCO 30013 – Accounting for Special Transactions

The Five-Step Model

The principles in the standard will be applied using a five-step model:

1. Step 1 - Identify the Contract with a Customer

A contract can be oral, written or implied by an entity’s business practice. A contract with
a customer will fall within the scope of PFRS 15 when all the following criteria are met:
• The parties to the contract have approved the contract;
• Each party’s rights in relation to the goods or services to be transferred can be
identified;
• The payment terms and conditions for the goods or services to be transferred can be
identified;
• The contract has commercial substance (i.e., the risk, timing or amount of the entity’s
future cash flows is expected to change as a result of the contract); and
• The collection of an amount of consideration to which the entity is entitled to in
exchange for the goods or services is probable.

If the above criteria are met, a contract shall not be re-assessed unless there is an
indication of a significant change in facts or circumstances, however if the contract does
not meet the above criteria the entity will continue to re-assess the contract going
forward to determine whether the criteria are subsequently met.

Is collectability probable?

• Assessment is done from the perspective of whether the customer has the “ability
and intention” to pay the amount of consideration to which the company expects to
be entitled.
• If this criterion is not met, then revenue cannot be recognized (effectively precluding
the use of the cash basis of accounting) and any consideration received is recorded
as a liability (e.g., unearned revenue) until either of the following events has
occurred:
✓ The entity has no remaining performance obligations and all or substantially all
the consideration for the performance obligations in the contract has been
received and is non-refundable.
✓ The contract is terminated and the consideration received is non-refundable.
COLLEGE OF ACCOUNTANCY AND FINANCE Page 41
ACCO 30013 – Accounting for Special Transactions

2. Step 2 - Identify the Performance Obligations in the Contract

Performance obligation - a promised good or service (i.e., promise in a contract) that is


distinct.

The good or service is distinct, if “both” of below criteria are met:


• The customer must be able to benefit from the good or service either on its own or
together with other readily available resources (i.e., capable of being distinct).
• The good or service is separately identifiable from other goods or services in the
contract (i.e., distinct within the context of the contract).

Separate Performance Obligation (SPO) should be separately identified if:


• The entity is not using the good or service as an input to produce the output specified
by the customer.
• The good or service does not significantly modify or customize another good or
service promised in the contract.
• The good or service is not highly dependent on, or highly interrelated with, other
goods or services promised in the contract.

Example: Identifying Distinct Goods

A bus manufacturer offers for sale both fully manufactured buses and spare parts. When
a customer purchases a fully manufactured bus, it is made up of many inputs (e.g.,
engine, tires, frame, etc.); however, the customer is not receiving these individual inputs
(e.g., the engine in isolation). Rather, these inputs are used to produce a combined
output to the customer (i.e., a bus).

Although engines may be sold separately as a spare part, which would indicate that it
meets the first criterion of being a distinct good (i.e., capable of being distinct), in the
context of a contract for a fully manufactured bus the engine is not considered distinct
because it is an input and the entity is providing a significant service by integrating the
engine with other goods and services in the contract (i.e., the engine is not distinct within
the context of the contract). Therefore in this type of contract, the engine is not
considered to be a distinct good or service, and is not a separate performance obligation.

Example: Series of Distinct Goods

A manufacturer enters into a contract with a customer to provide a series of similar


customized goods in a large quantity that will be delivered consecutively over time.

Based on the contract, the customer has title to and controls the work in progress as the
products are being manufactured. The manufacturer has determined an expected
average cost for manufacturing the product and has also determined that using an input
method based on costs (for this particular contract) is an appropriate method because it
represents the manufacturer’s performance when transferring control of the goods.

This contract is determined to be for a series of distinct goods that are effectively one
performance obligation because of the following:
• The goods are similar in nature and are transferred consecutively over time.
• The contract is satisfied over time because the customer has title to and controls the
work in progress while the products are being manufactured and therefore meets the
criterion for over-time recognition.
• The same method (i.e., cost to cost) is used to measure progress toward the
satisfaction of the performance obligation of each individual product.
COLLEGE OF ACCOUNTANCY AND FINANCE Page 42
ACCO 30013 – Accounting for Special Transactions

• Given that the series criterion is met, the entity must account for the series as one
performance obligation. This is not optional.

Example: Subcontracting Services

A building contractor enters into a contract with a customer to build a residential home.
Various goods and services are required throughout the construction process.

As is typical with construction projects, the various goods and services are subject to
significant integration such that the customer is in effect getting a single output and the
entire contract is viewed as one performance obligation.

Assume in this particular case, the builder is not completing all the required services
needed for the combined output to satisfy the customer on its own. The building
contractor sub-contracts out the electrical and plumbing work to an external, independent
contractor. Does contracting out some of the services change the assessment such that
the contract is no longer one performance obligation? Does the ability to sub-contract
services mean the services are distinct performance obligations?

In this case, although the electrical and plumbing services are being obtained elsewhere
(i.e., capable of being distinct), the building contractor is still providing a level of
integration as the nature of the promise is to produce a combined output (i.e., a
residential home). The sub-contracting of services would not result in a change in the
assessment of whether a good or service is distinct.

Construction-type and production-type contracts involve the transfer to the customer of


many goods and services that are capable of being distinct (such as various building
materials, labour or services and project management services). However, identifying
each individual good or service as a promise would not faithfully represent the nature of
the entity’s promise to the customer nor result in a useful depiction of the entity’s
performance.

An entity that is a principal in a contract may satisfy a performance obligation by itself or


it may engage another party (for example, a subcontractor) to satisfy some or all of a
performance obligation on its behalf. When an entity that is a principal satisfies a
performance obligation, the entity recognizes revenue in the gross amount of
consideration to which it expects to be entitled in exchange for those goods or services
transferred.

Therefore, as long as the entity remains a principal, engaging another party to transfer
some goods or services, does not change the identification of performance obligations.

3. Step 3 - Determine the Transaction Price

Transaction price – the amount of consideration to which an entity expects to be


entitled in exchange for transferring promised goods or services to a customer, excluding
amounts collected on behalf of third parties.

Variable Consideration - encompasses any amount that is variable under a contract,


(for example: performance bonuses, penalties, discounts, rebates, refunds, credits, price
concessions, incentives and the customer’s right to return products).

PAS 11/PAS 18 (old) PFRS 15 (new) Impact


• Revenue is recognized at the “fair • PFRS 15 prescribes “expected “Highly probable”
value of the consideration value” or “most likely amount” for is the new threshold.
COLLEGE OF ACCOUNTANCY AND FINANCE Page 43
ACCO 30013 – Accounting for Special Transactions

receivable”. variable consideration.


• Revenue is recognized if it is • Only permits variable amounts to
“probable” that the revenue will flow be included to the extent that it is
to the entity and that the amount “highly probable that the revenue
can be “measured reliably”. will not reverse”

Example: Volume Discount


On January 1, 20XX, an entity (with a December 31 year-end) enters into a contract with
a customer for the sale of a product for total consideration of P100/unit for a one-year
period. If the customer purchases more than 75 units of this product, the total
consideration will be P80/unit retrospectively applied (i.e., all purchases will be at
P80/unit). Initially, the entity does not believe the customer will purchase more than 75
units. However, on May 20, 20XX, the entity infers from the customer’s purchasing
pattern that the customer will in fact meet this target. Assume that the purchase pattern
is as follows:
January - 10 units
February - 15 units
March - 15 units
May 20 - 12 units

Because the total contract consideration of P100/unit includes a fixed component (i.e.,
P80/unit) and a variable component (i.e., P20/unit), the variable component must be
assessed to determine the estimated value and whether this amount should be
constrained. Consider the following:

Contract Inception
Based on past history (that has predictive value) with this particular product and the
customer, the entity does not believe the customer will meet the target required for the
consideration to drop to P80/unit. The entity believes instead there is a high probability a
significant revenue reversal will not occur because the target expected purchases will not
exceed 75 units and therefore the entity expects to be entitled to consideration of
P100/unit rather than P80/unit. The entity records the total consideration amount of
P100/unit.

March 31, 20XX


If the conclusions reached at inception remain appropriate, revenue for the period ending
March 31, 20XX will be recognized using the P100/unit price.

May 20, 20XX


The entity now estimates that because of increased purchases the customer will likely
exceed the targeted 75 units. It is now highly probable a significant revenue reversal will
occur. Therefore revenue will have to be retrospectively adjusted to the P80/unit price.
This adjustment will be recorded in the current period, meaning May 20XX (i.e., in Q2).

Time Period Amount of Revenue to be Recognized


January – March P4,000 = (10+15+15 units) x P100/unit
P160 = (10+15+15 units) x (P80/unit-P100/unit) + (12 units) x
May 20
80/unit

Significant Financing Component

Revenue recognized should reflect the “cash selling price” of the particular good or
service at the time the good or service is transferred.
COLLEGE OF ACCOUNTANCY AND FINANCE Page 44
ACCO 30013 – Accounting for Special Transactions

Significant financing component is determined for:


a. the difference, if any, between the amount of promised consideration and the cash
selling price of the promised goods or services; and
b. the combined effect of both of the following:
• the expected length of time between when the entity transfers the promised
goods or services to the customer and when the customer pays for those goods
or services; and
• the prevailing interest rates in the relevant market.

However, entities may still encounter a situation in which both the above factors exist,
but nevertheless may conclude that a significant financing component does not exist. At
least one of the following factors must be present:
a. the customer paid for the goods or services in advance and the timing of the transfer
of those goods or services is at the discretion of the customer (for example, in the
telecom industry, when a customer purchases a prepaid phone card).
b. a substantial amount of the consideration promised by the customer is variable and
the amount or timing of that consideration varies on the basis of the occurrence or
non-occurrence of a future event that is not substantially within the control of the
customer (for example, if the consideration is a sales-based royalty).
c. the difference between the promised consideration and the cash selling price of the
good or service arises for reasons other than the provision of finance to either the
customer or the entity, and the difference between those amounts is proportional to
the reason for the difference (for example, the payment terms might provide the
entity or the customer with protection from the other party failing to adequately
complete some or all of its obligations under the contract).

As a practical expedient, an entity need not adjust the transaction price in a contract for
the effects of a significant financing component, if the period between when the customer
pays for the good or service and when the entity transfers the good or service is one
year or less.

Interest income or interest expense resulting from a significant financing component


should be presented separately from revenue from contracts with customers. An entity
might present interest income as revenue in circumstances in which interest income
represents income from an entity’s ordinary activities.

Example: Advance Payment and Assessment of Discount Rate

An entity enters into a contract with a customer to sell an asset. Control of the asset will
transfer to the customer in two years (i.e., the performance obligation will be satisfied at
a point in time). The contract includes two alternative payment options: payment of
P5,000 in two years when the customer obtains control of the asset or payment of
P4,000 when the contract is signed. The customer elects to pay P4,000 when the
contract is signed.

The entity concludes that the contract contains a significant financing component
because of the length of time between when the customer pays for the asset and when
the entity transfers the asset to the customer, as well as the prevailing interest rates in
the market.

The interest rate implicit in the transaction is 11.8%, which is the interest rate necessary
to make the two alternative payment options economically equivalent. However, the
entity determines that the rate that should be used in adjusting the promised
consideration is 6%, which is the entity’s incremental borrowing rate.
COLLEGE OF ACCOUNTANCY AND FINANCE Page 45
ACCO 30013 – Accounting for Special Transactions

The following journal entries illustrate how to account for the significant financing
component:
1. Recognize a contract liability for the P4,000 payment received at contract inception:
Cash 4,000
Contract liability 4,000

2. During the two years from contract inception until the transfer of the asset, the entity
adjusts the promised amount of consideration (in accordance with paragraph 65 of
PFRS 15) and accretes the contract liability by recognizing interest on P4,000 at 6%
for two years:
Interest expense 494
Contract liability 494
*Interest Expense Year I = P4,000 × 6% = P240
*Interest Expense Year II = P4,240 × 6% = P254

3. Recognize revenue for the transfer of the asset:


Contract liability 4,494
Revenue 4,494
**P4,494 = P4,000 + (6% interest per year for two years).

Example: Determining If a Significant Financing Component Exists

A consulting engineering firm enters into a two-year contract to provide consulting


services to a builder for electrical work in a new commercial complex. As part of the
contract, the builder will make payments to the consultant at certain milestones in the
contract. The engineering firm has concluded this contract is one performance obligation
to be satisfied over time. The milestone payments have been scheduled to coincide with
performance. 10% of each milestone payment will be withheld by the builder and paid
once all consulting work is completed and all electrical work in the complex is
operational.

Although there is effectively a delay between performance and payment (i.e., related to
the holdback), the delay is for reasons other than financing (i.e., the holdback is intended
to protect the builder from the engineering firm failing to complete its obligations under
the contract) and the amount of the holdback is proportionate to this intended reason.

4. Step 4 - Allocate the Transaction Price to the Performance Obligations in the


Contract

Transaction price (at inception) should be allocated to all the performance obligations in
the contract in proportion to the stand-alone selling prices of each performance
obligation.

Suitable methods for estimating the stand-alone selling price include (but are not limited
to) the:
• adjusted market assessment approach
• expected cost-plus-a-margin approach and,
• in limited circumstances, the residual approach.

PAS 18 (old) PFRS 15 (new) Impact


• Does not give any guidance on how to • Specific guidance on Profit reporting
identify the separately identifiable allocation of patterns will change.
components of a single transaction and how transaction price
to allocate selling price and as a result, there based on stand-alone
COLLEGE OF ACCOUNTANCY AND FINANCE Page 46
ACCO 30013 – Accounting for Special Transactions

were different practices applied. prices.


• For example, telco companies recognized
revenue from the sale of monthly plans in full
as the service was provided, and no revenue
for handset – they treated the cost of handset
as the cost of acquiring the customer.

Examples: Allocating the Transaction Price

1. A retailer sells a customer a computer-and-printer package for P900. The retailer has
determined that these are two separate performance obligations and regularly sells
the printer for P300 and the computer for P700. In this case, the entity will allocate
the P900 total transaction price as follows:
• Printer: P270 = P900 × (P300 / (P300 + P700))
• Computer: P630 = P900 × (P700 / (P300 + P700))

2. On July 1, 20X1, Johnny enters into a 12-month telecom plan with the local mobile
operator ABC with terms:
• Johnny’s monthly fixed fee is P100.
• Johnny receives a free handset at the inception of the plan.

ABC sells the same handsets for P300 and the same monthly prepayment plans
w/out handset for P80/month.

Step 1: Identify the contract, which is the 12-month plan with Johnny.
Step 2: Identify all performance obligations from the contract with Johnny:
• Obligation to deliver a handset
• Obligation to deliver network services over 1 year
Step 3: The transaction price is P1,200, calculated as monthly fee of P100 x 12
months.
Step 4: Allocate the transaction price:
PO SOP % Amount
Handset 300.00 23.80% 285.60
Network service 960.00 76.20% 914.40
Total 1,260.00 100.00% 1,200.00
Step 5: Recognize the revenue when ABC satisfies the performance obligations.
• When ABC gives a handset to Johnny, it needs to recognize the revenue
of P285.60;
• When ABC provides network services to Johnny, it needs to recognize the
total revenue of P914.40.

The journal entries are summarized as follows:


Unbilled receivable 285.60
Revenue from sale of handset 285.60
To record sale of handset.

Accounts receivable 100.00


Revenue from network services 76.20
Unbilled receivable 23.80
To record network services.

Profits reported on Year 20X1:


PO PAS 18 (old) PFRS 15 (new)
Handset 0.00 285.60
COLLEGE OF ACCOUNTANCY AND FINANCE Page 47
ACCO 30013 – Accounting for Special Transactions

Network service 600.00 (100 x 6) 457.20 (76.20 x 6)


Total 600.00 742.80

5. Step 5 - Recognize Revenue as the Entity Satisfies a Performance Obligation

PFRS 15 is based on the “transfer of control” as opposed to the transfer of risks and
rewards.

Control - is the ability to direct the use of, and obtain substantially all of, the remaining
benefits associated with the asset. However, control can also be an ability to prevent
other entities from directing the use of, and obtaining the benefits from, an asset.

PAS 11 (old) PFRS 15 (new) Impact


• An entity recognizes • If none of the three (3) criteria to • For transactions
revenue and profits recognize revenue over time are met, then currently accounted for
over time by reference the entity recognizes revenue when it under the percentage of
to the stage of transfers control of the good or service to completion method, it
completion. the customer, which may be until practical will be necessary for
completion. management to
evaluate contracts
against the new criteria
to establish whether it is
appropriate to recognize
revenue over time or at
a point in time.
• Example of acceptable • An entity measures progress to • The contractor needs to
methods to measure satisfaction of the performance obligations determine whether an
stage of completion using a method that depicts performance. input or output method
are: This may be either: appropriately depicts its
-Contract costs -An input method (e.g., contract costs performance under the
COLLEGE OF ACCOUNTANCY AND FINANCE Page 48
ACCO 30013 – Accounting for Special Transactions

incurred to date as incurred to date as a percentage of total contract.


percentage of total forecast costs); or
forecast costs -An output method (e.g., surveys of work
-Surveys of work completed to date).
performed; or • A contractor applying the input method
-Physical completion. excludes the effect of any inputs that do
not depict its performance in transferring
control of goods or services to the
customer. For example, when using cost-
to-cost method, the contractor would
exclude unexpected amounts of wastage
materials, labour and any uninstalled
materials.

Two Transition Options

1. Full Retrospective
• Any completed contracts that begin and end within the same annual reporting period
do not need to be restated. Assuming a calendar year-end entity, if a contract began
on January 15, 2018, and ended on November 30, 2018, it would not need to be
restated when the fiscal 2018 figures are being restated.
• For any completed contracts that included variable consideration, entities may use
the transaction price on the contract completion date rather than estimating variable
consideration amounts in the prior reporting periods. Effectively, this permits entities
to use hindsight in estimating variable consideration and should simplify transition
because it will reduce the amount of information required for transition and reduce
efforts by not requiring entities to determine the transaction price at the end of each
period.
• For all periods presented in the financial statements prior to the initial date of
application, the disclosures of the amount of total consideration allocated to
remaining performance obligations (and the required explanation of when it is
expected to be recognized) do not need to be included. Entities must apply the use of
practical expedients consistently to contracts with similar characteristics and in
similar circumstances.

2. Modified Retrospective - rather than restating the comparative year (e.g., fiscal 2018),
entities can recognize the cumulative effects of applying PFRS 15 as an adjustment to
opening retained earnings in the period of initial application (i.e., January 1, 2018, for a
calendar-year-end entity).

Presentation

Contract Asset is an entity’s right to consideration in exchange for goods or services the
entity has transferred to a customer that is conditional on something other than the passage
of time (e.g., the performance of another obligation). If the right to this consideration is
unconditional and only the passage of time is required before the consideration is due, then
a receivable exists and a contract asset does not exist.

A customer’s right to a refund does not make such consideration conditional because future
refunds do not affect an entity’s present right to obtain consideration. Such obligations
should be treated as a separate refund liability.

Both receivables and contract assets are subject to credit risk; however, contract assets are
also subject to other types of risks, such as performance risk. Impairment losses relating to a
customer’s credit risk (i.e., impairment of a contract asset or receivable) are measured
based on the guidance in PFRS 9 Financial Instruments.
COLLEGE OF ACCOUNTANCY AND FINANCE Page 49
ACCO 30013 – Accounting for Special Transactions

*Note: term “contract asset” does not necessarily need to be used and entities may change
the terminology.

Bad Debts are recognized as a result of customer credit risk. However, this is not a credit
risk existing at the inception of the contract and therefore is different from the evaluation of
credit risk completed at inception (i.e., collectability threshold assessed as part of Step 1 of
the PFRS 15 revenue model).

Impairment losses (which may include bad debts) should be recognized as an expense.
Gross revenue is not adjusted for these amounts since revenue should be recognized at the
amount at which the entity expects to be entitled (and not the amount it receives). This
amount should not be adjusted by amounts the entity was not able to collect.

Additionally, the portion of the impairment losses recognized that relate to short-term
receivables from contracts with customers (i.e., effectively bad debts) should be disclosed
separately in the financial statements notes (i.e., separate from other impairment losses),
since this provides users with information on receivables management (i.e., bad debts).

Disclosure

1. Contracts with Customers


• Amount of revenue recognized (which must be disclosed separately from other
revenue sources)
COLLEGE OF ACCOUNTANCY AND FINANCE Page 50
ACCO 30013 – Accounting for Special Transactions

• Impairment losses recognized on any receivables or contract assets (If these


amounts are already presented in the Statement of Comprehensive Income, the
above additional disclosures are not required).

2. Significant Judgments used to determine:


• Timing of satisfaction of performance obligations
• Transaction price and amounts allocated to performance obligations.

3. Assets Recognized from the Costs to Obtain or Fulfil a Contract with a Customer
• Judgments required to determine the amount of the costs incurred to obtain or fulfil a
contract with a customer
• Method used for amortization
• Closing balances of these assets by main category
• Amount of amortization and impairment (if any) recognized during the period

4. Practical expedients used.

PROBLEMS

1. AAA Company entered into a contract with a private company on July 1, 2020. The
private company subscribes for a monthly plan for 12 months and in return, the private
company receives free handset. The private company will pay monthly fee of P1,600 and
it gets the handset immediately after signing the contract. AAA sells the same handset
for P4,800 and the same monthly plans for P1,280 without the handset.

Determine the following:


a. Revenue to be recognized on 2020 and 2021
b. Entries to record the transactions for 2020 and 2021

2. BBB Company was organized in 2020. The firm entered into a contract with a private
company to do the following: bookkeeping activities, preparation of monthly and annual
financial statements and annual tax returns. The annual fee is P100,000 for bookkeeping
services, P5,000 for monthly reports and P50,000 for annual report and tax return.

Determine the following:


a. Revenue to be recognized on 2020
b. Entries to record the transactions

3. CCC Company sells their air-conditioning units to various customers. In its contract with
Makati Hotel, Carrier Philippines promised to deliver 25 split-type air-conditioning units
for a total price of P800,000 by the end of 2020. The contract with Makati Hotel contains
a provision for free repairs and maintenance services within 2 years from the date of
purchase. All the 25 units were delivered and Makati Hotel paid P800,000.

Determine the following:


a. Revenue to be recognized on 2020
b. Entries to record the transactions

4. On January 1, 2020, DDD Company entered into a contract with a customer to build an
item of specialized equipment for delivery on April 30, 2020. However, the exact delivery
date is hard to estimate. The amount of consideration specified in the contract is
P300,000, but the customer is willing to pay a higher price if delivered earlier (i.e.,
amount will be increased or decreased by P500 for each day, depending on whether the
actual delivery date is before or after April 30, 2020).
COLLEGE OF ACCOUNTANCY AND FINANCE Page 51
ACCO 30013 – Accounting for Special Transactions

Below is the sensitivity analysis:


• 20% likelihood that product will be delivered on April 30, 2020;
• 50% likelihood that product will be delivered 30 days before April 30, 2020; and
• 30% likelihood that product will be delivered 30 days after April 30, 2020.

How much is transaction price for this contract?

5. On November 10, 2020, EEE Company enters into a contract with a private company to
deliver 2 Dining Set, 2 Sofa and 5 Cabinets for a lumpsum price of P500,000. It is
stipulated in the contract that payment is made only upon delivery of all the orders,
though the items may be delivered in installment. The stand-alone prices of the items
are: Dining Set – P80,000 per set; Sofa – P100,000 per set; and Cabinet – 40,000 per
piece. On December 20, 2020, the following were delivered: 1 Dining Set, 1 Sofa and 3
Cabinets. The remaining items were delivered on January 15, 2021.

Determine the following:


a. How many performance obligations are there in the contract?
b. How much is revenue to be recognized in 2020 and 2021?
c. Entries to record the transactions in 2020 and 2021.
COLLEGE OF ACCOUNTANCY AND FINANCE Page 52
ACCO 30013 – Accounting for Special Transactions

REVENUE RECOGNITION: CONSIGNMENT SALES

Learning Objectives

After discussing the topics, the learner will be able to:


• Understand the nature of consignment sales;
• Journalize the required entries in the books of the consignor and consignee; and
• Compute remittance from the consignee, net income resulting from consignment
transactions and value of merchandise in the hands of the consignee.

Consignment

It is an arrangement whereby the owner or seller (consignor) delivers or transfers


merchandise to another party (consignee) who acts as an agent for the consignor to sell the
goods.

Common Practices:
1. No revenue is recognized by the consignor until the goods are sold by the consignee to
outside or third parties.
2. Upon shipment of the merchandise to the consignee, a special inventory account is
established on the consignor’s books to identify the consigned merchandise.
3. Consignment expenses paid by the consignor are added to the inventory cost:
• Freight
• Insurance of the goods shipped
• Handling and cartage fee
*Sometimes, the consignee paid the above on behalf of the consignor (for
reimbursement).
4. Any reimbursable expenses paid by the consignee are considered selling expenses:
• Advertising costs
• Delivery and Installation
5. When a sale is made, the consignor recognizes the revenue only upon notification of
sale by the consignee accompanied by the remittance of the cash due supported by an
account sales.
6. Commission on units sold is also treated as deduction from sales revenue in determining
the net income from the shipment.

Proforma Entries:
Transaction Consignor Consignee
Inventory on Consignment xxx
Shipment of Goods Memorandum Entry
Inventory xxx
Payment of Freight by Inventory on Consignment xxx
No Entry
consignor Cash xxx
Cash advance by the Cash xxx Receivable from Consignor xxx
consignee Payable to Consignee xxx Cash xxx
Inventory xxx
Return of Goods from
Delivery Expense xxx Memorandum Entry
consignee to consignor
Inventory on Consignment xxx
Delivery Fee paid by
Delivery Expense xxx Receivable from Consignor xxx
consignee on the
Payable to Consignee xxx Cash xxx
returned goods
Payment of Advertising
Receivable from Consignor xxx
and Selling Cost paid by No Entry until notified by consignee
Cash xxx
consignee
Cash xxx
Sale of Merchandise No Entry until notified by consignee
Payable to Consignor xxx
COLLEGE OF ACCOUNTANCY AND FINANCE Page 53
ACCO 30013 – Accounting for Special Transactions

Cash xxx
Payable to Consignor xxx
Notification of sales, Advertising Fee xxx
Receivable from Consignor xxx
expenses and remittance Commission Expense xxx
Commission Revenue xxx
of cash Payable to Consignee xxx
Cash xxx
Consignment Sales xxx
Adjustment of Cost of Cost of Sales xxx
No Entry
Sales Inventory on Consignment xxx

Note: In cases where goods are returned by the consignee, the inventoriable cost allocated
to these units is charged to the units sold (period cost).

Important Notes to Remember:


1. Amount assigned to unsold units in the hand of the consignee represents the original
cost of the inventory plus inventoriable costs incurred in the shipment.
2. The amount of costs and expenses assigned to sold units represents the cost of goods
sold (original cost), inventoriable cost (freight on shipment), and expenses incurred in the
return of goods to consignor.
3. Freight paid by the consignee representing delivery charges for goods delivered to
customers is treated as selling costs (period cost).

PROBLEMS

1. AAA Company shipped 400 electronic gadgets to SM Company at a suggested retail


price of P650 each. The consignee paid for the shipping costs of P3,000 on the entire
shipment. The two parties agreed that any sale in excess of the suggested retail price
will accrue to the consignee. SM submitted an account sale of 300 gadgets, 55% of
which are sold at P650 each and the rest at P800. The consignee has a mark-up on cost
of 25%.

Determine the following:


a. Cash Remittance
b. Consignment Income/(Loss)
c. Cost of Ending Inventory
d. Entries to record the transactions in both books.

2. BBB Company consigned 1,000 units of ordinary printer, costing P700 each, to SM
Company to be sold at P1,200 per unit. The agreement calls for 20% commission on
units sold and any expenses paid by SM Company is reimbursable by the consignor.
The consignor paid P25,000 freight and insurance of P10,000 for the shipment. At the
end of the month, SM Company reported that 190 units are still on hand. It remitted
P743,000 after deducting commission of P192,000 and advertising cost of P25,000.

Determine the following:


a. Number of units sold
b. Consignment Income/(Loss)
c. Cost of Ending Inventory
d. Entries to record the transactions in both books

3. CCC Company shipped 150 oven costing P9,000 each on consignment basis to SM
Aura to be sold at P15,000 each. The consignee is to be allowed a commission of 15%.
CCC incurred P15,000 in shipping the 150 units. The agreement requires that SM Aura
will advance 50% of the cost of the oven, to be applied to periodic remittances in
proportion to the units sold. Any expenses related to the consigned units incurred by SM
Aura are also deductible from the remittance. On October 31, SM Aura rendered an
account sale that includes the following deductions:
COLLEGE OF ACCOUNTANCY AND FINANCE Page 54
ACCO 30013 – Accounting for Special Transactions

• Advertising costs – P15,000


• Delivery Expense – P350 per unit
• Commission – P146,250

Determine the following:


a. Number of units sold
b. Cost of Ending Inventory
c. Cash Remittance
d. Consignment Income/(Loss)
e. Entries to record the transactions in both books

4. DDD Company agrees to transfer television sets to SM Company on a consignment


basis. The consignee is to sell a set at 40% gross profit and is to receive a 15%
commission on sales price. The company account all the inventoriable costs before
setting the selling price. The consignor agrees to reimburse the consignee for all
expenses related to the consignment. The agreement also calls for an advance payment
by the consignee of 20% per set based on selling price; the said advance is to be
deducted as settlement is made for each set sold and returned. The consignee is to
provide an account sale quarterly and is to make cash remittance for the amount owed at
that time. The following consignment sales activities occurred during the last quarter of
the current year:
• Set shipped – 100
• Manufacturing cost per set – P12,000
• Freight charges on the shipment paid by the consignor – P60,000
• The consignee made advance payments on the sets received
• Advertising cost paid by the consignee – P30,000

The consignee sold 80 sets for cash; expenses of delivery and installation were P20,000.
After notifying the consignor with the total sets sold for the period, the consignee
returned 10 sets and paid corresponding freight charges of P5,000 on the return.

Determine the following:


a. Net Income (Loss)
b. Cash Remittance
c. Ending Inventory
d. Entries to record the transactions in both books

5. EEE Company consigned 2,000 units of digital camera, costing P800 each to SM
Megamall to be sold at P1,500 per unit. The agreement calls for 15% commission on
units sold and any expenses paid by the consignee is reimbursable by the consignor.
EEE paid P40,000 freight and insurance of P20,000 for the shipment. At the end of the
month, SM remitted the amount due to the consignor after deducting commission of
P360,000 and advertising costs of P50,000. Record shows that 250 units were left to the
consignee and some units were returned to the consignor. SM paid the cost of shipment
of the returned units amounting to P20,000.

Determine the following:


a. Net Income (Loss)
b. Cash Remittance
c. Ending Inventory
d. Entries to record the transactions in both books
COLLEGE OF ACCOUNTANCY AND FINANCE Page 55
ACCO 30013 – Accounting for Special Transactions

REVENUE RECOGNITION: INSTALLMENT SALES

Learning Objectives

After discussing the topics, the learner will be able to:


• Understand the nature of installment sales;
• Journalize the required entries to record the installment sales transactions;
• Be able to account for the effect of repossession;
• Be able to account for the effect of trade-in; and
• Compute the revenue from installment sales.

Types of Sale

There are two types of sales in the ordinary course of business:


1. Regular Sales – either cash or credit sales. Revenue is recognized at the time of sale,
the point at which goods or services are delivered/performed in exchange for a
consideration.
2. Installment Sales – any type of sale on which the payment is in periodic installment
over a period of time.

Installment Sales

In installment sales, the cash inflow is deferred. The general provision of PAS 18 provides
that revenue shall be measured at the fair value of the consideration received or
receivable. Revenue shall be recognized when all of the following conditions have been
satisfied:
1. The entity has transferred to the buyer the significant risks and rewards of ownership of
the goods.
2. The entity retains neither continuing managerial involvement to the degree usually
associated with ownership nor effective control over the goods sold.
3. The amount of revenue can be measured reliably.
4. It is probable that the economic benefits associated with the transactions will flow to the
entity.
5. The costs incurred or to be incurred in respect of the transaction can be measured
reliably.

When the cash inflow is deferred, the fair value of the consideration is determined by
discounting all future receipts using an imputed rate of interest. The difference between the
fair value and the nominal amount of the consideration is recognized as interest revenue.

Proforma Entries (if balance is interest-bearing):


Cash xxx
Installment Sales Receivable xxx
Installment Sales xxx
To record the sale.

Cost of Installment Sales xxx


Inventory/Shipments on Installment Sales xxx
To record cost of installment.

Cash xxx
Interest Revenue xxx
Installment Sales Receivable xxx
To record collection.
COLLEGE OF ACCOUNTANCY AND FINANCE Page 56
ACCO 30013 – Accounting for Special Transactions

Proforma Entries (if balance is non-interest bearing):


Cash xxx
Installment Sales Receivable xxx
Installment Sales xxx
Discount on Installment Receivable xxx
To record the sale.

Cost of Installment Sales xxx


Inventory/Shipments on Installment Sales xxx
To record cost of installment.

Cash xxx
Discount on Installment Receivable xxx
Installment Receivable xxx
Interest Revenue xxx
To record collection.

Defaulted or Repossessed Accounts

When a customer fails to make any further payment, the seller repossesses the
merchandise or property sold. The repossessed merchandise is recorded at its market value
(Net Realizable Value) or its unrecovered cost whichever is lower. Net Realizable Value is
the estimated selling price in the ordinary course of business less the sum of estimated costs
of reconditioning, the estimated costs necessary to make the sale and normal profit. The net
realizable value is then compared with the unrecovered cost to determine the gain or loss on
repossession:
• If NRV of the merchandise is equal to its unrecovered cost, no gain or loss on
repossession.
• If NRV is less than its unrecovered cost, there is a loss on repossession.
• If NRV is greater than unrecovered cost, there is a gain on repossession, but the gain
is deferred until the merchandise is sold. In this case, the repossessed item is recorded
at its unrecovered cost.

Proforma Entries:
Repossession xxx
Loss on Repossession xxx
Installment Receivable xxx
To record repossession with loss.

Repossession xxx
Gain on Repossession xxx
Installment Receivable xxx
To record repossession with loss.

Trade In

Sometimes, companies accept second-hand item as initial payment for the item sold and this
is measured at its Net Realizable Value (NRV). Net Realizable Value is the estimated selling
price in the ordinary course of business less reconditioning cost, cost to sell and normal
profit. An allowance is normally allowed on the trade in item and this amount is compared
with its net realizable value to arrive at over allowance or under allowance on trade in.
• If NRV is equal to allowance on trade in, there is no over or under allowance on trade
in.
• If NRV is greater than the allowance on trade in, there is under allowance. Under
allowance on trade in is treated as addition to installment sales.
COLLEGE OF ACCOUNTANCY AND FINANCE Page 57
ACCO 30013 – Accounting for Special Transactions

• If NRV is less than the allowance in trade in, there is over allowance. Over allowance
on trade in is treated as deduction from installment sales.

*The over or under allowance is adjusted to the original sales price to arrive at adjusted
sales.

Proforma Entries
Cash (downpayment) xxx
Trade-in xxx
Installment Receivable xxx
Installment Sales xxx
Discount on Installment Sales xxx
To record installment sales.

PROBLEMS

1. On January 1, 2020, AAA Company sold a luxury car costing P1,080,000 for an
installment price of P2,200,000 collectible as follows: 20% down payment, the balance is
payable in 5 equal annual installments of P352,000 starting December 31, 2018 and the
imputed rate of interest is 10%.

Determine the following:


a. Installment Sales Revenue to be recognized in 2020
b. Net income to be recognized in 2020
c. Entries to record the transactions for 2020

2. On October 1, 2020, BBB Company sold for P1,400,000 an equipment costing


P910,000. A down payment of P400,000 was received with the balance payable on a
monthly basis of P100,462 for one year starting October 31, 2020. Interest is to be
charged at a monthly rate of 3% on the unpaid balance, the monthly installment is to be
applied first to the interest and the balance to the principal.

Determine the following:


a. Installment Sales Revenue to be recognized in 2020
b. Net income to be recognized in 2020
c. Entries to record the transactions for 2020

3. On March 2, 2020, CCC Company sold Innova car costing P576,000 for P960,000. In
2020, a total of P130,000 is collected on the contract of which P30,000 represents
finance charges. After making religious payments, the customer defaulted and the
vehicle was repossessed. CCC estimated that the market value of the repossessed
vehicle is 80% of its unrecovered cost.

Determine the following:


a. Gain or Loss on Repossession
b. Balance of Repossessed Item Account
c. Net Income to be recognized
d. Entries to record the transactions

4. DDD Company sold vehicles on installment basis. On October 1, 2020, Kia Model
costing P747,600 was sold for P1,236,000 on installment basis. A cash down payment of
P216,000 was made. In addition to the cash down payment, a Ford Model was accepted
with trade in value of P300,000 and balance is payable in 12 monthly installments
amounting to P78,000 due starting at the end of October. The Ford Model has an
estimated selling price of P 400,000 after reconditioning the same for P50,000. The
COLLEGE OF ACCOUNTANCY AND FINANCE Page 58
ACCO 30013 – Accounting for Special Transactions

normal gross profit, to be considered in the fair value computation, is 10%. DDD already
adapted PFRS 15 in recognizing revenue. The market rate of interest is 12%.

Determine the following:


a. Trade In to be reported on December 31, 2020
b. Installment Sales
c. Net Income in 2020
e. Entries to record the transactions in 2020

5. EEE Company sells new automobiles. A new Escapade costing P700,000 was sold on
October 1, 2020 for P1,060,000. A used Toyota Revo was accepted as down payment
and an allowance of P300,000 was accepted on the trade in. The balance is payable in
36 equal monthly installments of P25,500 starting November 1, 2020. The interest rate is
10%. EEE anticipates selling price on reconditioned automobile of P350,000 after
reconditioning it for P50,000. Selling costs equal to 5% of selling price is also anticipated.
The gross profit for reconditioned automobile is expected to be 20%.

Determine the following:


a. Revenue to be recognized in 2020
b. Balance of Trade in Item Account
c. Net income to be recognized in 2020
d. Entries to record the transactions in 2020
COLLEGE OF ACCOUNTANCY AND FINANCE Page 59
ACCO 30013 – Accounting for Special Transactions

REVENUE RECOGNITION: FRANCHISING

Learning Objectives

After discussing the topics, the learner will be able to:


• Understand the nature of franchising;
• Journalize the required entries to record the franchise transactions;
• Compare and journalize the different methods of accounting for initial franchise fee and
the related cost of franchise revenues – prior to substantial performance and upon
substantial performance of service; and
• Compute the revenue from franchising.

Franchising

It refers to the method of practicing and using another's perfected business concept. In a
franchise relationship, the franchisee is granted the right to market a product or a service
under a marketing plan or a system that uses the trademark, name, logo and advertising
owned by the franchisor.

Common Services Rendered by Franchisor


1. Conduct of Market Studies
2. Training of Employees
3. Finding suitable location
4. Sale of products as operation goes on

Major Problems in Accounting for Franchising


1. Initial franchise fee received by the franchisor
2. Cost incurred prior to opening of the franchise outlet
3. Payment of the continuing franchise fee
4. Sale of goods by the franchisor to franchisee

Rules to follow in Accounting for Franchise


1. If the probability of refunding the initial franchise fee is extremely low, the amount of
future services to be rendered to the franchisee is minimal, collectivity of the note is
reasonably assured and substantial performance has occurred, the total franchise fee
(the down payment and the present value of the note) is recorded as REVENUE
immediately.
2. If the initial down payment is not refundable and represents a fair measure of the
services already provided, with a significant amount of services still to be performed by
the franchisor in future periods and the collectivity of the note is reasonably assured, the
down payment is recorded as REVENUE and the present value of the note is recorded
as UNEARNED/DEFERRED REVENUE.
3. If the initial down payment is not refundable and no future services are required by the
franchisor, but collection of the note is uncertain, the down payment is recorded as
REVENUE and the note may not be recorded.
4. If the initial down payment is refundable or substantial services are yet to be performed
and the collection of the note is uncertain, the down payment is recorded as
UNEARNED/DEFERRED REVENUE and the note may not be recorded.

Proforma Entries (if balance is interest-bearing):


Cash xxx
Franchise Fee Receivable xxx
Franchise Fee Revenue xxx
To record the franchise.
COLLEGE OF ACCOUNTANCY AND FINANCE Page 60
ACCO 30013 – Accounting for Special Transactions

Cost of Franchise xxx


Cash xxx
To record cost of franchise.

Cash xxx
Interest Revenue xxx
Franchise Fee Receivable xxx
To record collection.

Proforma Entries (if balance is non-interest bearing):


Cash xxx
Franchise Fee Receivable xxx
Franchise Fee Revenue xxx
Discount on Franchise Receivable xxx
To record the franchise.

Cost of Installment Sales xxx


Cash xxx
To record cost of franchise.

Cash xxx
Discount on Franchise Receivable xxx
Franchise Fee Receivable xxx
Interest Revenue xxx
To record collection.

Repossession of Franchise

If the franchisor repossesses its franchise, it may or may not refund the consideration
previously paid by the franchisee. If the franchise fee is refunded, the accounting treatment
is equivalent to a cancellation of sales. The revenue previously recognized is reported as a
reduction of the revenue of the current period in which the franchise is reacquired.

If the franchise fee is not refunded, no adjustment is recorded. However, if a balance is still
owed by the franchisee, the Allowance for Uncollectible Accounts related to the transaction
should be reviewed. In addition, any deferred revenue on the original sale should now be
recognized in full.

PROBLEMS

1. AAA Company, a franchisor, charges an initial franchise fee of P750,000 payable


P500,000 cash upon signing the franchise agreement and the balance, evidenced by a
non-interest-bearing note, is payable in 5 semi-annual installments of P50,000. The
franchisor agrees to conduct a market studies, to find a suitable location and to train
employees. On July 1, 2020, an agreement was signed between AAA and BBB
Company, a franchisee. Total cost incurred by the franchisor in the performance of the
services as of December 31, 2020 is P150,000. The first installment on the note was
made on December 31, 2020. Additional cost incurred in January 2021 is P75,000. The
franchise outlet opened on February 15, 2021. The management of BBB estimated that
they can borrow at the rate of 10%.

Determine the following:


a. Revenue to be recognized in 2020
b. Income to be recognized in 2020
c. Entries to record the transactions in 2020
COLLEGE OF ACCOUNTANCY AND FINANCE Page 61
ACCO 30013 – Accounting for Special Transactions

2. Use the same information in Problem 1. Determine the revenue to be recognized and the
entries to record the transactions assuming:
a. The probability of refunding the initial franchise fee is extremely low, the amount of
future services to be rendered to the franchisee is minimal, collectivity of the note is
reasonably assured and substantial performance has occurred.
b. The initial down payment is not refundable and represents a fair measure of the
services already provided, with a significant amount of services still to be performed
by the franchisor in future periods and the collectivity of the note is reasonably
assured.
c. The initial down payment is not refundable and no future services are required by the
franchisor, but collection of the note is uncertain.
d. The initial down payment is refundable or substantial services are yet to be
performed and the collection of the note is uncertain.

3. CCC Company entered into a franchise agreement with DDD Company on January 1,
2020. The initial franchise fee agreed upon is P950,000, of which, P150,000 is payable
upon signing and the balance to be covered by a non-interest-bearing note payable in
four equal annual installments to be paid starting December 31, 2020. It was agreed that
the down payment is not refundable and royalties equal to 5% of its average sales every
calendar year must be paid to CCC.

The following expenses were incurred:


Initial services: Continuing services:
Direct cost - 331,200 Direct cost - 66,240
Indirect cost - 41,400 Indirect cost - 16,560

The management of DDD estimated that they can borrow at the rate of 5%. The
franchisee commenced its operations on June 30, 2020. DDD reported monthly average
sales of P80,000 since the start of its operations.

Determine the following:


a. Revenue to be recognized in 2020
b. Unearned revenue to be recognized in 2020
c. Income to be recognized in 2020
d. Entries to record the transactions in 2020

4. EEE Company charges new franchisees an initial franchise fee of P2,500,000. Of this
amount, P1,000,000 is payable in cash when the agreement is signed, and the
remainder is to be paid in three equal annual installments which are evidenced by 12%
interest-bearing note. In consideration therefore, EEE Company will assist in locating
business site, conduct a market study to estimate earnings potential, supervise
construction of a building, and provide initial training to employees. On December 1,
2020, EEE entered into a franchising agreement with FFF Company. By the end of the
year, EEE had completed all of the initial services required at a cost of P800,000 and it
has ascertained that the collection of the note is reasonably assured.

Determine the following:


a. Revenue to be recognized in 2020
b. Unearned revenue to be recognized in 2020
c. Income to be recognized in 2020
d. Entries to record the transactions in 2020

5. On March 1, 2020, GGG Company entered into a franchise contract with HHH Company.
The franchise agreement required the franchisee, HHH, to pay a non-refundable upfront
fee in the amount of P1,440,000 and on-going payment of royalty equivalent to 5% of the
COLLEGE OF ACCOUNTANCY AND FINANCE Page 62
ACCO 30013 – Accounting for Special Transactions

sales of the franchisee. In relation to the non-refundable upfront fee, the franchise
agreement required GGG to render the following performance obligations to HHH:
• To construct the franchisee’s stall with stand-alone selling price of P300,000.
• To supply cooking equipment and cash registers. Price of competitors for the similar
items (cooking equipment and cash registers) is valued at P240,000 while the
forecast of the expected cost of GGG for the performance obligation is P200,000 plus
an appropriate margin above cost of 25%.
• To deliver 10,000 units of raw materials with stand-alone selling price of P460,000.
• To allow HHH to use the entity’s trade name for a period of 10 years starting on the
inception of the contract with stand-alone selling price of P600,000.

During 2020, GGG satisfied its performance obligations to supply cooking equipment
and install cash registers, constructed the franchisee’s stall and was able to deliver 6,000
units of raw materials to HHH. Also, during 2020, the franchisee reported sales revenue
amounting to P720,000. GGG incurred a total cost of P500,000 related to franchise. The
entity had determined that the performance obligations are separate and distinct from
one another and accounted under PFRS 15.

Determine the following:


a. Revenue to be recognized in 2020
b. Income to be recognized in 2020
c. Entries to record the transactions in 2020
COLLEGE OF ACCOUNTANCY AND FINANCE Page 63
ACCO 30013 – Accounting for Special Transactions

REVENUE RECOGNITION: LONG-TERM CONSTRUCTION CONTRACT

Learning Objectives

After discussing the topics, the learner will be able to:


• Define different terms or accounts used in construction contracts;
• Apply PFRS 15 in accounting for long-term construction contracts;
• Determine the percentage of completion of a contract;
• Compute and journalize the transactions related to construction revenue, costs, and
gross profit (loss);
• Journalize billings and collections with mobilization fee, retention fee and penalty clause;
and
• Account for construction contracts with modification clause.

Longterm Construction Contract

The transactions of a contractor (construction firm) such as acquisition of plant assets,


materials and payment of operating expenses are accounted for in the same manner as
manufacturing and merchandising business. Specific transactions related to the construction
activities include:
• Incurrence of materials, labor and overhead costs
• Billings to customer
• Collections from customer
• Recognition of revenue, cost of revenue and gross profit

The main issue in accounting for long-term construction contracts is the timing of recognition
of revenue, the related costs of revenue and gross profit.

Methods in Measuring Progress


• Output Method – It recognizes revenue on the basis of direct measurement of the value
to the customer of the goods or services transferred to date relative to the remaining
goods or services promised under the contract. Examples are surveys of performance
completed to date, appraisals of results achieved, milestones reached and units
produced or units delivered.
• Input Method – It recognizes revenue on the basis of entity’s efforts or inputs to the
satisfaction of a performance obligation relative to the total expected inputs to the
satisfaction of the performance obligation. Examples are resources consumed, labor
hours expended, costs incurred and machine hours used.

Terms to Remember
1. Contract Revenue – is the initial amount of revenue agreed in the contract and
variations in contract work, claims and incentive payments.
2. Contract Costs – are costs incurred in the construction of the project such as:
• Site labor costs including site supervision
• Cost of materials used in construction
• Depreciation of plant and equipment used on the contract
• Cost of moving plant, equipment and materials to and from the contract site
• Cost of hiring plant and equipment
• Cost of design and technical assistance that is directly related to the contract
• Estimated cost of rectification and guarantee work including expected warranty costs
• Claims from third parties
• Insurance (allocated)
COLLEGE OF ACCOUNTANCY AND FINANCE Page 64
ACCO 30013 – Accounting for Special Transactions

• Cost of design and technical assistance that is not directly related to the contract
(allocated)
• Construction overheads such as cost of preparing and processing construction
personnel payroll (allocated)
• Other costs specifically chargeable to the customer under the terms of the contract
3. Contact Price – is the price agreed upon between the contractor and the customer for
the conduction of a specific project.
4. Cost Incurred to Date – is the cumulative cost of construction incurred by the contractor
from the time the project started up to a particular date.
5. Estimated Costs to Complete – it is the additional cost of construction reasonably
expected to be incurred to complete the project.
6. Total Estimated Cost (Cost at Completion) – it is the sum of the costs incurred to date
and the estimated costs to complete.
7. Total Estimated Gross Profit – it is the excess of the contract price over the total
estimated cost.
8. Total Estimated (Anticipated) Loss – it the excess of the total estimated costs over the
contract price.
9. Mobilization Fee – are indirect costs of construction that includes all costs associated in
relocating manpower to the project site, transporting tools and equipment, establishment
of offices, etc.
10. Retention Fee – it is the amount withheld by the customer as a security until the
completion of the project. It is based on the work completed usually ranging from 5% to
10%.

Important Notes:
1. Under the cost-to-cost method, construction revenue, construction cost and income are
recognized as follows:
• Construction Revenue = Contract Price x Percentage of Completion
• Construction Cost = Actual Costs Incurred during the year
• Income = Construction Revenue – Construction Cost
2. When the outcome of a construction contract cannot be estimated reliably, revenue
should be recognized only to the extent of contract costs incurred that is probable will be
recoverable. Hence, no profit is to be recognized.
3. When it is probable that the contract costs will exceed total contract revenue (anticipated
loss), the expected loss should be recognized immediately.

Proforma Entries:
Construction Materials xxx
Cash/Accounts Payable xxx
To record purchase of construction materials.

Construction in Progress xxx


Construction Materials xxx
To record cost of materials incurred in construction.

Construction in Progress xxx


Payroll xxx
To record cost of construction labor.

Construction in Progress xxx


Cash/Accounts Payable xxx
To record other construction overhead costs.

Construction Costs xxx


Construction in Progress xxx
COLLEGE OF ACCOUNTANCY AND FINANCE Page 65
ACCO 30013 – Accounting for Special Transactions

Construction Revenue xxx


To record revenue, cost and income.

Construction Costs xxx


Construction Revenue xxx
Construction in Progress xxx
To record revenue, cost and loss.

Accounts Receivable xxx


Progress Billings on Construction Contracts xxx
To record billings to customers.

Cash xxx
Accounts Receivable xxx
To record collections from customers.

Progress Billings on Construction Contracts xxx


Construction in Progress xxx
To record turnover of the project.

PROBLEMS

1. AAA Company has a contract to build a building. The initial amount of revenue agreed in
the contract is P9,000,000. The contractor’s initial estimate of contract costs is
P8,000,000. It will take 3 years to build the building. By the end of year 1, the
contractor’s estimate of contract costs has increased to P8,050,000.

In year 2, the customer approves a variation resulting in an increase in the contract


revenue of P200,000 and estimated additional contract costs of P150,00. At the end of
year 2, cost incurred include P100,000 for standard materials stored at the site to be
used in year 3 to complete the project. The contractor determines the percentage of
completion by calculating the proportion that contract costs incurred for work performed
to date to the estimated total contract costs.

A summary of financial data during the construction period is as follows:


Year 1 Year 2 Year 3
Contract costs incurred 2,093,000 4,075,000 2,032,000
Estimated costs to complete 5,957,000 2,032,000 0
Billings to client 2,700,000 4,500,000 2,000,000
Collections 2,500,000 4,000,000 2,700,000

Determine the following:


a. Percentage of completion for year 1 and year 2
b. Amount of contract revenue, contract costs and profit to appear in the income
statement of each year
c. Entries required to record the transactions each year

2. BBB Company entered into a contract with Hometown Hospital on July 1, 2018 to
construct a four-storey hospital. At that time, BBB estimated that it would take 3-4 years
to complete the project. The total contract is P25,000,000. In 2018, the outcome of the
project cannot be estimated reliably but in 2019, BBB was already certain the project is
estimated to cost P20,000,000. The building was substantially completed in 2020.

Contract costs incurred, estimated costs to complete and progress billings were as
follows:
COLLEGE OF ACCOUNTANCY AND FINANCE Page 66
ACCO 30013 – Accounting for Special Transactions

2018 2019 2020


Contract costs incurred 3,250,000 5,750,000 8,550,000
Estimated costs to complete 13,000,000 9,000,000 1,950,000
Billings to client 2,500,000 6,500,000 9,500,000

Determine the following:


a. Percentage of completion each year
b. Amount of contract revenue, contract costs and profit to appear in the income
statement of each year
c. Entries required to record the transactions each year

3. CCC Company began operations on January 1, 2019. During the year, Andrei entered
into a contract with DDD Company to construct a manufacturing facility. At that time,
CCC estimated that it would take 3 years to complete the facility at a total cost of
P4,500,000. The total contract price is P6,300,000. In 2019, CCC incurred P1,185,800 in
construction costs and estimated that P4,204,200 more will be needed for its completion.
DDD Company was billed and paid 30% of the contract price.

The following year, CCC incurred P3,299,200 and the engineer’s estimate of cost to
complete is P2,500,000. It is expected that the project will be completed this year.

Determine the following:


a. Percentage of completion each year
b. Amount of contract revenue, contract costs and profit to appear in the income
statement of each year
c. Entries required to record the transactions each year

4. EEE Company has the following data for the year just ended:
Costs to Date – P9,800,000
Estimated Cost at Completion – P12,250,000
Contract Price – P17,500,000

The company accounts for its large jobs by the percentage of completion method.
Billings are done as follows:
• 20% down payment upon contract signing as mobilization fee
• Amount billed to client is according to percentage of completion
• 10% retention fee deductible at the last billing

Determine the following:


a. Entries to record the transactions
b. Amount of contract revenue, contract costs and profit to appear in the income
statement
c. Balance of Construction in Progress and Progress Billings

5. FFF Company has accepted a project of constructing a 20-storey condominium in BGC


at a fixed price contract of P60,000,000. The project is expected to be completed in three
years. The details of the costs incurred in the 1st year are:
• Cost of labor in the site – P2,500,000
• Cost of construction materials, including P500,000 of special materials to be used in
Year 2 – P12,000,000
• Depreciation of construction equipment and other overhead in the site – P2,800,000
• Depreciation of idle construction equipment – P900,000
• Paid and incurred rectification work – P260,000
COLLEGE OF ACCOUNTANCY AND FINANCE Page 67
ACCO 30013 – Accounting for Special Transactions

• Miscellaneous expenses incurred by brokers and sales agents in pre-selling the units
– P1,500,000
• Total estimated costs at completion – P48,000,000

Determine the following:


a. Contract revenue
b. Cost of revenue
c. Gross profit (loss) to be recognized
COLLEGE OF ACCOUNTANCY AND FINANCE Page 68
ACCO 30013 – Accounting for Special Transactions

REFERENCES

Baysa, Gloria & Lupisan, Ma, Concepcion (2011). Advanced Accounting Part 1. Sta. Mesa,
Manila

Dayag, Antonio (2019). Advanced Financial Accounting and Reporting. GIC Enterprise & Co.
Inc., Sampaloc, Manila

Dayag, Antonio (2018). Advanced Financial Accounting Volume 1. Lajara Publishing House,
Sta. Cruz, Manila

International Accounting Standards Board. IFRS 15: Revenue from Contract with
Customers. Retrieved from: https://www.ifrs.org/issued-standards/list-of-
standards/ifrs-15-revenue-from-contracts-with-customers/

Rante, Gloria, et al. (2020). Accounting for Special Transactions. Millenium Books, Inc.,
Mandaluyong City

You might also like